Что раньше делается умножение или деление: Порядок выполнения действий в выражениях без скобок и со скобками

Содержание

Знаки указывающие порядок выполнения действий. Изучение правил порядка действий

Начальная школа подходит к концу, скоро ребёнок шагнёт в углубленный мир математики. Но уже в этот период школьник сталкивается с трудностями науки. Выполняя простое задание, ребёнок путается, теряется, что в результате приводит к отрицательной отметке за выполненную работу. Чтобы избежать подобных неприятностей, нужно при решении примеров, уметь ориентироваться в порядке, по которому нужно решать пример. Не верно распределив действия, ребёнок не правильно выполняет задание. В статье раскрываются основные правила решения примеров, содержащих в себе весь спектр математических вычислений, включая скобки. Порядок действий в математике 4 класс правила и примеры.

Перед выполнением задания попросите своё чадо пронумеровать действия, которые он собирается выполнить. Если возникли затруднения – помогите.

Некоторые правила, которые необходимо соблюдать при решении примеров без скобок:

Если в задании необходимо выполнить ряд действий, нужно сначала выполнить деление или умножение, затем . Все действия выполняются по ходу письма. В противном случае, результат решения будет не верным.

Если в примере требуется выполнить , выполняем по порядку, слева направо.

27-5+15=37 (при решении примера руководствуемся правилом. Сначала выполняем вычитание, затем – сложение).

Научите ребёнка всегда планировать и нумеровать выполняемые действия.

Ответы на каждое решённое действие записываются над примером. Так ребёнку гораздо легче будет ориентироваться в действиях.

Рассмотрим ещё один вариант, где необходимо распределить действия по порядку:

Как видим, при решении соблюдено правило, сначала ищем произведение, после — разность.

Это простые примеры, при решении которых, необходима внимательность. Многие дети впадают в ступор при виде задания, в котором присутствует не только умножение и деление, но и скобки. У школьника, не знающего порядок выполнения действий, возникают вопросы, которые мешают выполнить задание.

Как говорилось в правиле, сначала найдём произведение или частное, а потом всё остальное. Но тут же есть скобки! Как поступить в этом случае?

Решение примеров со скобками

Разберём конкретный пример:

  • При выполнении данного задания, сначала найдём значение выражения, заключённого в скобки.
  • Начать следует с умножения, далее – сложение.
  • После того, как выражение в скобках решено, приступаем к действиям вне их.
  • По правилам порядка действий, следующим шагом будет умножение.
  • Завершающим этапом станет .

Как видим на наглядном примере, все действия пронумерованы. Для закрепления темы предложите ребёнку решить самостоятельно несколько примеров:

Порядок, по которому следует вычислять значение выражения уже расставлен. Ребёнку останется только выполнить непосредственно решение.

Усложним задачу. Пусть ребёнок найдёт значение выражений самостоятельно.

7*3-5*4+(20-19) 14+2*3-(13-9)
17+2*5+(28-2) 5*3+15-(2-1*2)
24-3*2-(56-4*3) 14+12-3*(21-7)

Приучите ребёнка решать все задания в черновом варианте. В таком случае, у школьника будет возможность исправить не верное решение или помарки. В рабочей тетради исправления не допустимы. Выполняя самостоятельно задания, дети видят свои ошибки.

Родители, в свою очередь, должны обратить внимание на ошибки, помочь ребёнку разобраться и исправить их. Не стоит нагружать мозг школьника большими объёмами заданий. Такими действиями вы отобьёте стремление ребёнка к знаниям. Во всём должно быть чувство меры.

Делайте перерыв. Ребёнок должен отвлекаться и отдыхать от занятий. Главное помнить, что не все обладают математическим складом ума. Может из вашего ребёнка вырастет знаменитый философ.

На данном уроке подробно рассмотрен порядок выполнения арифметических действий в выражениях без скобок и со скобками. Учащимся предоставляется возможность в ходе выполнения заданий определить, зависит ли значение выражений от порядка выполнения арифметических действий, узнать отличается ли порядок арифметических действий в выражениях без скобок и со скобками, потренироваться в применении изученного правила, найти и исправить ошибки, допущенные при определении порядка действий.

В жизни мы постоянно выполняем какие-либо действия: гуляем, учимся, читаем, пишем, считаем, улыбаемся, ссоримся и миримся. Эти действия мы выполняем в разном порядке. Иногда их можно поменять местами, а иногда нет. Например, собираясь утром в школу, можно сначала сделать зарядку, затем заправить постель, а можно наоборот. Но нельзя сначала уйти в школу, а потом надеть одежду.

А в математике обязательно ли выполнять арифметические действия в определенном порядке?

Давайте проверим

Сравним выражения:
8-3+4 и 8-3+4

Видим, что оба выражения совершенно одинаковы.

Выполним действия в одном выражения слева направо, а в другом справа налево. Числами можно проставить порядок выполнения действий (рис. 1).

Рис. 1. Порядок действий

В первом выражении мы сначала выполним действие вычитания, а затем к результату прибавим число 4.

Во втором выражении сначала найдем значение суммы, а потом из 8 вычтем полученный результат 7.

Видим, что значения выражений получаются разные.

Сделаем вывод: порядок выполнения арифметических действий менять нельзя .

Узнаем правило выполнения арифметических действий в выражениях без скобок.

Если в выражение без скобок входят только сложение и вычитание или только умножение и деление, то действия выполняют в том порядке, в каком они написаны.

Потренируемся.

Рассмотрим выражение

В этом выражении имеются только действия сложения и вычитания. Эти действия называют

действиями первой ступени .

Выполняем действия слева направо по порядку (рис. 2).

Рис. 2. Порядок действий

Рассмотрим второе выражение

В этом выражении имеются только действия умножения и деления — это действия второй ступени.

Выполняем действия слева направо по порядку (рис. 3).

Рис. 3. Порядок действий

В каком порядке выполняются арифметические действия, если в выражении имеются не только действия сложения и вычитания, но и умножения и деления?

Если в выражение без скобок входят не только действия сложения и вычитания, но и умножения и деления, или оба этих действия, то сначала выполняют по порядку (слева направо) умножение и деление, а затем сложение и вычитание.

Рассмотрим выражение.

Рассуждаем так. В этом выражении имеются действия сложения и вычитания, умножения и деления. Действуем по правилу. Сначала выполняем по порядку (слева направо) умножение и деление, а затем сложение и вычитание. Расставим порядок действий.

Вычислим значение выражения.

18:2-2*3+12:3=9-6+4=3+4=7

В каком порядке выполняются арифметические действия, если в выражении имеются скобки?

Если в выражении имеются скобки, то сначала вычисляют значение выражений в скобках.

Рассмотрим выражение.

30 + 6 * (13 — 9)

Мы видим, что в этом выражении имеется действие в скобках, значит, это действие выполним первым, затем по порядку умножение и сложение. Расставим порядок действий.

30 + 6 * (13 — 9)

Вычислим значение выражения.

30+6*(13-9)=30+6*4=30+24=54

Как нужно рассуждать, чтобы правильно установить порядок арифметических действий в числовом выражении?

Прежде чем приступить к вычислениям, надо рассмотреть выражение (выяснить, есть ли в нём скобки, какие действия в нём имеются) и только после этого выполнять действия в следующем порядке:

1. действия, записанные в скобках;

2. умножение и деление;

3. сложение и вычитание.

Схема поможет запомнить это несложное правило (рис. 4).

Рис. 4. Порядок действий

Потренируемся.

Рассмотрим выражения, установим порядок действий и выполним вычисления.

43 — (20 — 7) +15

32 + 9 * (19 — 16)

Будем действовать по правилу. В выражении 43 — (20 — 7) +15 имеются действия в скобках, а также действия сложения и вычитания. Установим порядок действий. Первым действием выполним действие в скобках, а затем по порядку слева направо вычитание и сложение.

43 — (20 — 7) +15 =43 — 13 +15 = 30 + 15 = 45

В выражении 32 + 9 * (19 — 16) имеются действия в скобках, а также действия умножения и сложения. По правилу первым выполним действие в скобках, затем умножение (число 9 умножаем на результат, полученный при вычитании) и сложение.

32 + 9 * (19 — 16) =32 + 9 * 3 = 32 + 27 = 59

В выражении 2*9-18:3 отсутствуют скобки, зато имеются действия умножения, деления и вычитания. Действуем по правилу. Сначала выполним слева направо умножение и деление, а затем от результата, полученного при умножении, вычтем результат, полученный при делении. То есть первое действие — умножение, второе — деление, третье — вычитание.

2*9-18:3=18-6=12

Узнаем, правильно ли определен порядок действий в следующих выражениях.

37 + 9 — 6: 2 * 3 =

18: (11 — 5) + 47=

7 * 3 — (16 + 4)=

Рассуждаем так.

37 + 9 — 6: 2 * 3 =

В этом выражении скобки отсутствуют, значит, сначала выполняем слева направо умножение или деление, затем сложение или вычитание. В данном выражении первое действие — деление, второе — умножение. Третье действие должно быть сложение, четвертое — вычитание. Вывод: порядок действий определен верно.

Найдем значение данного выражения.

37+9-6:2*3 =37+9-3*3=37+9-9=46-9=37

Продолжаем рассуждать.

Во втором выражении имеются скобки, значит, сначала выполняем действие в скобках, затем слева направо умножение или деление, сложение или вычитание. Проверяем: первое действие — в скобках, второе — деление, третье — сложение. Вывод: порядок действий определен неверно. Исправим ошибки, найдем значение выражения.

18:(11-5)+47=18:6+47=3+47=50

В этом выражении также имеются скобки, значит, сначала выполняем действие в скобках, затем слева направо умножение или деление, сложение или вычитание. Проверяем: первое действие — в скобках, второе — умножение, третье — вычитание. Вывод: порядок действий определен неверно. Исправим ошибки, найдем значение выражения.

7*3-(16+4)=7*3-20=21-20=1

Выполним задание.

Расставим порядок действий в выражении, используя изученное правило (рис. 5).

Рис. 5. Порядок действий

Мы не видим числовых значений, поэтому не сможем найти значение выражений, однако потренируемся применять изученное правило.

Действуем по алгоритму.

В первом выражении имеются скобки, значит, первое действие в скобках. Затем слева направо умножение и деление, потом слева направо вычитание и сложение.

Во втором выражении также имеются скобки, значит, первое действие выполняем в скобках. После этого слева направо умножение и деление, после этого — вычитание.

Проверим себя (рис. 6).

Рис. 6. Порядок действий

Сегодня на уроке мы познакомились с правилом порядка выполнения действий в выражениях без скобок и со скобками.

Список литературы

  1. М.И. Моро, М.А. Бантова и др. Математика: Учебник. 3 класс: в 2-х частях, часть 1. — М.: «Просвещение», 2012.
  2. М.И. Моро, М.А. Бантова и др. Математика: Учебник. 3 класс: в 2-х частях, часть 2. — М.: «Просвещение», 2012.
  3. М.И. Моро. Уроки математики: Методические рекомендации для учителя. 3 класс. — М.: Просвещение, 2012.
  4. Нормативно-правовой документ. Контроль и оценка результатов обучения. — М.: «Просвещение», 2011.
  5. «Школа России»: Программы для начальной школы. — М.: «Просвещение», 2011.
  6. С.И. Волкова. Математика: Проверочные работы. 3 класс. — М.: Просвещение, 2012.
  7. В.Н. Рудницкая. Тесты. — М.: «Экзамен», 2012.
  1. Festival.1september.ru ().
  2. Sosnovoborsk-soobchestva.ru ().
  3. Openclass.ru ().

Домашнее задание

1. Определи порядок действий в данных выражениях. Найди значение выражений.

2. Определи, в каком выражении такой порядок выполнения действий:

1. умножение; 2. деление;. 3. сложение; 4. вычитание; 5. сложение. Найди значение данного выражения.

3. Составь три выражения, в которых такой порядок выполнения действий:

1. умножение; 2. сложение; 3. вычитание

1. сложение; 2. вычитание; 3. сложение

1. умножение; 2. деление; 3. сложение

Найди значение этих выражений.

В пятом веке до нашей эры древнегреческий философ Зенон Элейский сформулировал свои знаменитые апории, самой известной из которых является апория «Ахиллес и черепаха». Вот как она звучит:

Допустим, Ахиллес бежит в десять раз быстрее, чем черепаха, и находится позади неё на расстоянии в тысячу шагов. За то время, за которое Ахиллес пробежит это расстояние, черепаха в ту же сторону проползёт сто шагов. Когда Ахиллес пробежит сто шагов, черепаха проползёт ещё десять шагов, и так далее. Процесс будет продолжаться до бесконечности, Ахиллес так никогда и не догонит черепаху.

Это рассуждение стало логическим шоком для всех последующих поколений. Аристотель, Диоген, Кант, Гегель, Гильберт… Все они так или иначе рассматривали апории Зенона. Шок оказался настолько сильным, что «… дискуссии продолжаются и в настоящее время, прийти к общему мнению о сущности парадоксов научному сообществу пока не удалось… к исследованию вопроса привлекались математический анализ, теория множеств, новые физические и философские подходы; ни один из них не стал общепризнанным решением вопроса… » [Википедия, » Апории Зенона «]. Все понимают, что их дурят, но никто не понимает, в чем заключается обман.

С точки зрения математики, Зенон в своей апории наглядно продемонстрировал переход от величины к . Этот переход подразумевает применение вместо постоянных. Насколько я понимаю, математический аппарат применения переменных единиц измерения либо ещё не разработан, либо его не применяли к апории Зенона. Применение же нашей обычной логики приводит нас в ловушку. Мы, по инерции мышления, применяем постоянные единицы измерения времени к обратной величине. С физической точки зрения это выглядит, как замедление времени до его полной остановки в момент, когда Ахиллес поравняется с черепахой. Если время останавливается, Ахиллес уже не может перегнать черепаху.

Если перевернуть привычную нам логику, всё становится на свои места. Ахиллес бежит с постоянной скоростью. Каждый последующий отрезок его пути в десять раз короче предыдущего. Соответственно, и время, затрачиваемое на его преодоление, в десять раз меньше предыдущего. Если применять понятие «бесконечность» в этой ситуации, то правильно будет говорить «Ахиллес бесконечно быстро догонит черепаху».

Как избежать этой логической ловушки? Оставаться в постоянных единицах измерения времени и не переходить к обратным величинам. На языке Зенона это выглядит так:

За то время, за которое Ахиллес пробежит тысячу шагов, черепаха в ту же сторону проползёт сто шагов. За следующий интервал времени, равный первому, Ахиллес пробежит ещё тысячу шагов, а черепаха проползет сто шагов. Теперь Ахиллес на восемьсот шагов опережает черепаху.

Этот подход адекватно описывает реальность без всяких логических парадоксов. Но это не полное решение проблемы. На Зеноновскую апорию «Ахиллес и черепаха» очень похоже утверждение Эйнштейна о непреодолимости скорости света. Эту проблему нам ещё предстоит изучить, переосмыслить и решить. И решение нужно искать не в бесконечно больших числах, а в единицах измерения.

Другая интересная апория Зенона повествует о летящей стреле:

Летящая стрела неподвижна, так как в каждый момент времени она покоится, а поскольку она покоится в каждый момент времени, то она покоится всегда.

В этой апории логический парадокс преодолевается очень просто — достаточно уточнить, что в каждый момент времени летящая стрела покоится в разных точках пространства, что, собственно, и является движением. Здесь нужно отметить другой момент. По одной фотографии автомобиля на дороге невозможно определить ни факт его движения, ни расстояние до него. Для определения факта движения автомобиля нужны две фотографии, сделанные из одной точки в разные моменты времени, но по ним нельзя определить расстояние. Для определения расстояния до автомобиля нужны две фотографии, сделанные из разных точек пространства в один момент времени, но по ним нельзя определить факт движения (естественно, ещё нужны дополнительные данные для расчетов, тригонометрия вам в помощь). На что я хочу обратить особое внимание, так это на то, что две точки во времени и две точки в пространстве — это разные вещи, которые не стоит путать, ведь они предоставляют разные возможности для исследования.

среда, 4 июля 2018 г.

Очень хорошо различия между множеством и мультимножеством описаны в Википедии . Смотрим.

Как видите, «во множестве не может быть двух идентичных элементов», но если идентичные элементы во множестве есть, такое множество называется «мультимножество». Подобную логику абсурда разумным существам не понять никогда. Это уровень говорящих попугаев и дрессированных обезьян, у которых разум отсутствует от слова «совсем». Математики выступают в роли обычных дрессировщиков, проповедуя нам свои абсурдные идеи.

Когда-то инженеры, построившие мост, во время испытаний моста находились в лодке под мостом. Если мост обрушивался, бездарный инженер погибал под обломками своего творения. Если мост выдерживал нагрузку, талантливый инженер строил другие мосты.

Как бы математики не прятались за фразой «чур, я в домике», точнее «математика изучает абстрактные понятия», есть одна пуповина, которая неразрывно связывает их с реальностью. Этой пуповиной являются деньги. Применим математическую теорию множеств к самим математикам.

Мы очень хорошо учили математику и сейчас сидим в кассе, выдаем зарплату. Вот приходит к нам математик за своими деньгами. Отсчитываем ему всю сумму и раскладываем у себя на столе на разные стопки, в которые складываем купюры одного достоинства. Затем берем с каждой стопки по одной купюре и вручаем математику его «математическое множество зарплаты». Поясняем математику, что остальные купюры он получит только тогда, когда докажет, что множество без одинаковых элементов не равно множеству с одинаковыми элементами. Вот здесь начнется самое интересное.

В первую очередь, сработает логика депутатов: «к другим это применять можно, ко мне — низьзя!». Дальше начнутся уверения нас в том, что на купюрах одинакового достоинства имеются разные номера купюр, а значит их нельзя считать одинаковыми элементами. Хорошо, отсчитываем зарплату монетами — на монетах нет номеров. Здесь математик начнет судорожно вспоминать физику: на разных монетах имеется разное количество грязи, кристаллическая структура и расположение атомов у каждой монеты уникально. ..

А теперь у меня самый интересный вопрос: где проходит та грань, за которой элементы мультимножества превращаются в элементы множества и наоборот? Такой грани не существует — всё решают шаманы, наука здесь и близко не валялась.

Вот смотрите. Мы отбираем футбольные стадионы с одинаковой площадью поля. Площадь полей одинакова — значит у нас получилось мультимножество. Но если рассматривать названия этих же стадионов — у нас получается множество, ведь названия разные. Как видите, один и тот же набор элементов одновременно является и множеством, и мультимножеством. Как правильно? А вот здесь математик-шаман-шуллер достает из рукава козырный туз и начинает нам рассказывать либо о множестве, либо о мультимножестве. В любом случае он убедит нас в своей правоте.

Чтобы понять, как современные шаманы оперируют теорией множеств, привязывая её к реальности, достаточно ответить на один вопрос: чем элементы одного множества отличаются от элементов другого множества? Я вам покажу, без всяких «мыслимое как не единое целое» или «не мыслимое как единое целое».

воскресенье, 18 марта 2018 г.

Сумма цифр числа — это пляска шаманов с бубном, которая к математике никакого отношения не имеет. Да, на уроках математики нас учат находить сумму цифр числа и пользоваться нею, но на то они и шаманы, чтобы обучать потомков своим навыкам и премудростям, иначе шаманы просто вымрут.

Вам нужны доказательства? Откройте Википедию и попробуйте найти страницу «Сумма цифр числа». Её не существует. Нет в математике формулы, по которой можно найти сумму цифр любого числа. Ведь цифры — это графические символы, при помощи которых мы записываем числа и на языке математики задача звучит так: «Найти сумму графических символов, изображающих любое число». Математики эту задачу решить не могут, а вот шаманы — элементарно.

Давайте разберемся, что и как мы делаем для того, чтобы найти сумму цифр заданного числа. И так, пусть у нас есть число 12345. Что нужно сделать для того, чтобы найти сумму цифр этого числа? Рассмотрим все шаги по порядку.

1. Записываем число на бумажке. Что же мы сделали? Мы преобразовали число в графический символ числа. Это не математическое действие.

2. Разрезаем одну полученную картинку на несколько картинок, содержащих отдельные цифры. Разрезание картинки — это не математическое действие.

3. Преобразовываем отдельные графические символы в числа. Это не математическое действие.

4. Складываем полученные числа. Вот это уже математика.

Сумма цифр числа 12345 равна 15. Вот такие вот «курсы кройки и шитья» от шаманов применяют математики. Но это ещё не всё.

С точки зрения математики не имеет значения, в какой системе счисления мы записываем число. Так вот, в разных системах счисления сумма цифр одного и того же числа будет разной. В математике система счисления указывается в виде нижнего индекса справа от числа. С большим числом 12345 я не хочу голову морочить, рассмотрим число 26 из статьи про . Запишем это число в двоичной, восьмеричной, десятичной и шестнадцатеричной системах счисления. Мы не будем рассматривать каждый шаг под микроскопом, это мы уже сделали. Посмотрим на результат.

Как видите, в разных системах счисления сумма цифр одного и того же числа получается разной. Подобный результат к математике никакого отношения не имеет. Это всё равно, что при определении площади прямоугольника в метрах и сантиметрах вы получали бы совершенно разные результаты.

Ноль во всех системах счисления выглядит одинаково и суммы цифр не имеет. Это ещё один аргумент в пользу того, что . Вопрос к математикам: как в математике обозначается то, что не является числом? Что, для математиков ничего, кроме чисел, не существует? Для шаманов я могу такое допустить, но для ученых — нет. Реальность состоит не только из чисел.

Полученный результат следует рассматривать как доказательство того, что системы счисления являются единицами измерения чисел. Ведь мы не можем сравнивать числа с разными единицами измерения. Если одни и те же действия с разными единицами измерения одной и той же величины приводят к разным результатам после их сравнения, значит это не имеет ничего общего с математикой.

Что же такое настоящая математика? Это когда результат математического действия не зависит от величины числа, применяемой единицы измерения и от того, кто это действие выполняет.

Открывает дверь и говорит:

Ой! А это разве не женский туалет?
— Девушка! Это лаборатория по изучению индефильной святости душ при вознесении на небеса! Нимб сверху и стрелочка вверх. Какой еще туалет?

Женский… Нимб сверху и стрелочка вниз — это мужской.

Если у вас перед глазами несколько раз в день мелькает вот такое вот произведение дизайнерского искусства,

Тогда не удивительно, что в своем автомобиле вы вдруг обнаруживаете странный значок:

Лично я делаю над собой усилие, чтобы в какающем человеке (одна картинка), увидеть минус четыре градуса (композиция из нескольких картинок: знак минус, цифра четыре, обозначение градусов). И я не считаю эту девушку дурой, не знающей физику. Просто у неё дугой стереотип восприятия графических образов. И математики нас этому постоянно учат. Вот пример.

1А — это не «минус четыре градуса» или «один а». Это «какающий человек» или число «двадцать шесть» в шестнадцатеричной системе счисления. Те люди, которые постоянно работают в этой системе счисления, автоматически воспринимают цифру и букву как один графический символ.

Когда мы работаем с различными выражениями, включающими в себя цифры, буквы и переменные, нам приходится выполнять большое количество арифметических действий. Когда мы делаем преобразование или вычисляем значение, очень важно соблюдать правильную очередность этих действий. Иначе говоря, арифметические действия имеют свой особый порядок выполнения.

Yandex.RTB R-A-339285-1

В этой статье мы расскажем, какие действия надо делать в первую очередь, а какие после. Для начала разберем несколько простых выражений, в которых есть только переменные или числовые значения, а также знаки деления, умножения, вычитания и сложения. Потом возьмем примеры со скобками и рассмотрим, в каком порядке следует вычислять их. В третьей части мы приведем нужный порядок преобразований и вычислений в тех примерах, которые включают в себя знаки корней, степеней и других функций.

Определение 1

В случае выражений без скобок порядок действий определяется однозначно:

  1. Все действия выполняются слева направо.
  2. В первую очередь мы выполняем деление и умножение, во вторую – вычитание и сложение.

Смысл этих правил легко уяснить. Традиционный порядок записи слева направо определяет основную последовательность вычислений, а необходимость сначала умножить или разделить объясняется самой сутью этих операций.

Возьмем для наглядности несколько задач. Мы использовали только самые простые числовые выражения, чтобы все вычисления можно было провести в уме. Так можно быстрее запомнить нужный порядок и быстро проверить результаты.

Пример 1

Условие: вычислите, сколько будет 7 − 3 + 6 .

Решение

В нашем выражении скобок нет, умножение и деление также отсутствуют, поэтому выполняем все действия в указанном порядке. Сначала вычитаем три из семи, затем прибавляем к остатку шесть и в итоге получаем десять. Вот запись всего решения:

7 − 3 + 6 = 4 + 6 = 10

Ответ: 7 − 3 + 6 = 10 .

Пример 2

Условие: в каком порядке нужно выполнять вычисления в выражении 6: 2 · 8: 3 ?

Решение

Чтобы дать ответ на этот вопрос, перечитаем правило для выражений без скобок, сформулированное нами до этого. У нас здесь есть только умножение и деление, значит, мы сохраняем записанный порядок вычислений и считаем последовательно слева направо.

Ответ: сначала выполняем деление шести на два, результат умножаем на восемь и получившееся в итоге число делим на три.

Пример 3

Условие: подсчитайте, сколько будет 17 − 5 · 6: 3 − 2 + 4: 2 .

Решение

Сначала определим верный порядок действий, поскольку у нас здесь есть все основные виды арифметических операций – сложение, вычитание, умножение, деление. Первым делом нам надо разделить и умножить. Эти действия не имеют приоритета друг перед другом, поэтому выполняем их в написанном порядке справа налево. То есть 5 надо умножить на 6 и получить 30 , потом 30 разделить на 3 и получить 10 . После этого делим 4 на 2 , это 2 . Подставим найденные значения в исходное выражение:

17 − 5 · 6: 3 − 2 + 4: 2 = 17 − 10 − 2 + 2

Здесь уже нет ни деления, ни умножения, поэтому делаем оставшиеся вычисления по порядку и получаем ответ:

17 − 10 − 2 + 2 = 7 − 2 + 2 = 5 + 2 = 7

Ответ: 17 − 5 · 6: 3 − 2 + 4: 2 = 7 .

Пока порядок выполнения действий не заучен твердо, можно ставить над знаками арифметических действий цифры, означающие порядок вычисления. Например, для задачи выше мы могли бы записать так:

Если у нас есть буквенные выражения, то с ними мы поступаем точно так же: сначала умножаем и делим, затем складываем и вычитаем.

Что такое действия первой и второй ступени

Иногда в справочниках все арифметические действия делят на действия первой и второй ступени. Сформулируем нужное определение.

К действиям первой ступени относятся вычитание и сложение, второй – умножение и деление.

Зная эти названия, мы можем записать данное ранее правило относительно порядка действий так:

Определение 2

В выражении, в котором нет скобок, сначала надо выполнить действия второй ступени в направлении слева направо, затем действия первой ступени (в том же направлении).

Порядок вычислений в выражениях со скобками

Скобки сами по себе являются знаком, который сообщает нам нужный порядок выполнения действий. В таком случае нужное правило можно записать так:

Определение 3

Если в выражении есть скобки, то первым делом выполняется действие в них, после чего мы умножаем и делим, а затем складываем и вычитаем по направлению слева направо.

Что касается самого выражения в скобках, его можно рассматривать в качестве составной части основного выражения. При подсчете значения выражения в скобках мы сохраняем все тот же известный нам порядок действий. Проиллюстрируем нашу мысль примером.

Пример 4

Условие: вычислите, сколько будет 5 + (7 − 2 · 3) · (6 − 4) : 2 .

Решение

В данном выражении есть скобки, поэтому начнем с них. Первым делом вычислим, сколько будет 7 − 2 · 3 . Здесь нам надо умножить 2 на 3 и вычесть результат из 7:

7 − 2 · 3 = 7 − 6 = 1

Считаем результат во вторых скобках. Там у нас всего одно действие: 6 − 4 = 2 .

Теперь нам нужно подставить получившиеся значения в первоначальное выражение:

5 + (7 − 2 · 3) · (6 − 4) : 2 = 5 + 1 · 2: 2

Начнем с умножения и деления, потом выполним вычитание и получим:

5 + 1 · 2: 2 = 5 + 2: 2 = 5 + 1 = 6

На этом вычисления можно закончить.

Ответ: 5 + (7 − 2 · 3) · (6 − 4) : 2 = 6 .

Не пугайтесь, если в условии у нас содержится выражение, в котором одни скобки заключают в себе другие. Нам надо только применять правило выше последовательно по отношению ко всем выражениям в скобках. Возьмем такую задачу.

Пример 5

Условие: вычислите, сколько будет 4 + (3 + 1 + 4 · (2 + 3)) .

Решение

У нас есть скобки в скобках. Начинаем с 3 + 1 + 4 · (2 + 3) , а именно с 2 + 3 . Это будет 5 . Значение надо будет подставить в выражение и подсчитать, что 3 + 1 + 4 · 5 . Мы помним, что сначала надо умножить, а потом сложить: 3 + 1 + 4 · 5 = 3 + 1 + 20 = 24 . Подставив найденные значения в исходное выражение, вычислим ответ: 4 + 24 = 28 .

Ответ: 4 + (3 + 1 + 4 · (2 + 3)) = 28 .

Иначе говоря, при вычислении значения выражения, включающего скобки в скобках, мы начинаем с внутренних скобок и продвигаемся к внешним.

Допустим, нам надо найти, сколько будет (4 + (4 + (4 − 6: 2)) − 1) − 1 . Начинаем с выражения во внутренних скобках. Поскольку 4 − 6: 2 = 4 − 3 = 1 , исходное выражение можно записать как (4 + (4 + 1) − 1) − 1 . Снова обращаемся к внутренним скобкам: 4 + 1 = 5 . Мы пришли к выражению (4 + 5 − 1) − 1 . Считаем 4 + 5 − 1 = 8 и в итоге получаем разность 8 — 1 , результатом которой будет 7 .

Порядок вычисления в выражениях со степенями, корнями, логарифмами и иными функциями

Если у нас в условии стоит выражение со степенью, корнем, логарифмом или тригонометрической функцией (синусом, косинусом, тангенсом и котангенсом) или иными функциями, то первым делом мы вычисляем значение функции. После этого мы действуем по правилам, указанным в предыдущих пунктах. Иначе говоря, функции по степени важности приравниваются к выражению, заключенному в скобки.

Разберем пример такого вычисления.

Пример 6

Условие: найдите, сколько будет (3 + 1) · 2 + 6 2: 3 − 7 .

Решение

У нас есть выражение со степенью, значение которого надо найти в первую очередь. Считаем: 6 2 = 36 . Теперь подставим результат в выражение, после чего оно примет вид (3 + 1) · 2 + 36: 3 − 7 .

(3 + 1) · 2 + 36: 3 − 7 = 4 · 2 + 36: 3 − 7 = 8 + 12 − 7 = 13

Ответ: (3 + 1) · 2 + 6 2: 3 − 7 = 13 .

В отдельной статье, посвященной вычислению значений выражений, мы приводим и другие, более сложные примеры подсчетов в случае выражений с корнями, степенью и др. Рекомендуем вам с ней ознакомиться.

Если вы заметили ошибку в тексте, пожалуйста, выделите её и нажмите Ctrl+Enter

Мы рассмотрим в этой статье три варианта примеров:

1. Примеры со скобками (действия сложения и вычитания)

2. Примеры со скобками (сложение, вычитание, умножение, деление)

3. Примеры, в которых много действий

1 Примеры со скобками (действия сложения и вычитания)

Рассмотрим три примера. В каждом из них порядок действий обозначен цифрами красного цвета:

Мы видим, что порядок действий в каждом примере будет разный, хотя числа и знаки одинаковые. Это происходит потому, что во втором и третьем примере есть скобки.

*Это правило для примеров без умножения и деления. Правила для примеров со скобками, включающих действия умножения и деления мы рассмотрим во второй части этой статьи.

Чтобы не запутаться в примере со скобками, можно превратить его в обычный пример, без скобок. Для этого результат, полученный в скобках, записываем над скобками, далее переписываем весь пример, записывая вместо скобок этот результат, и далее выполняем все действия по порядку, слева направо:

В несложных примерах можно все эти операции производить в уме. Главное — сначала выполнить действие в скобках и запомнить результат, а затем считать по порядку, слева направо.

А теперь — тренажеры!

1) Примеры со скобками в пределах до 20. Онлайн тренажер.

2) Примеры со скобками в пределах до 100. Онлайн тренажер.

3) Примеры со скобками. Тренажер №2

4) Вставь пропущенное число — примеры со скобками. Тренажер

2 Примеры со скобками (сложение, вычитание, умножение, деление)

Теперь рассмотрим примеры, в которых кроме сложения и вычитания есть умножение и деление.

Сначала рассмотрим примеры без скобок:

Есть одна хитрость, как не запутаться при решении примеров на порядок действий. Если нет скобок, то выполняем действия умножения и деления, далее переписываем пример, записывая вместо этих действий полученные результаты. Затем выполняем сложение и вычитание по порядку:

Если в примере есть скобки, то сначала нужно избавиться от скобок: переписать пример, записывая вместо скобок полученный в них результат. Затем нужно выделить мысленно части примера, разделенные знаками «+» и «-«, и посчитать каждую часть отдельно. Затем выполнить сложение и вычитание по порядку:

3 Примеры, в которых много действий

Если в примере много действий, то удобнее будет не расставлять порядок действий во всем примере, а выделить блоки, и решить каждый блок отдельно. Для этого находим свободные знаки «+» и «–» (свободные — значит не в скобках, на рисунке показаны стрелочками).

Табличка на двери

6/2(1+2) | Блог инженера

Вот сижу что-то ночью опять… Решил написать своё мнение о популярном сейчас вопросе: один или девять?

Я думаю, по изображению сверху стало уже понятно, о чём идёт речь. Знак умножения – он опущен перед скобками, и… как считать?

Посмотрим с двух позиций.

1) Знак умножения просто опущен. Тогда изначальная запись выражения выглядит так: .

Шесть делим на два, умножаем на сумму единицы и двойки и (всё просто супер, детка) получаем девять. Ответ – 9. Вроде всё красиво, но…

2) Знак умножения не просто опущен. Как так – не просто? А просто так и нельзя опустить. Итак, вот есть инфа, которую, похоже, взяли из учебника за седьмой класс (изначальный источник не найден, но нагуглил в методичке какого-то математического лицея):

Случаи возможного пропуска знака умножения:
1) между буквенными множителями;
2) между числовым и буквенным множителем;
3) между множителем и скобкой;
4) между выражениями в скобках.

Что это для нас значит? А то, что если знак умножения опустили так, как описано в предыдущем пункте, то поступили неправильно, потому что двойка в примере – не множитель перед скобкой, а просто один из трёх множителей (если рассматривать деление как частный случай умножения). Поэтому, если он опущен правильно, то имеем.

И это в том случае, если правило выше абсолютно точное. Но без конкретного источника (утверждается, что это школьный учебник) можно не рассчитывать на то, что оно точное. В школьной математике много требований, которыми даже в разделах вышки порой пренебрегают.

Это правило, к тому же, может оказаться неполным: вдруг нельзя опускать знак между скобкой и множителем в такой ситуации? Составлял бы я правила, я бы так и поступил. Спорная ситуация? Ставь ещё одну пару скобок! Будет вполне однозначно и всем понятно.

От себя скажу, что я часть после деления воспринимаю как нечто целое, т.е. скобку с множителем, мне это кажется вполне естественным. Почему же возникает спор? Многие запоминают, что «всегда можно опустить знак умножения». Но это не так. 2 умножить на 3 не есть 23, а произведение переменных c, o и s не всегда будет правильно понято.

На первый взгляд становится понятно, что человек, сказавший, что ответ – 1, просто забыл о порядке действий, его смутило отсутствие знака умножения. Здесь это чем-то напоминает мне загадку о ножках в комнате (где вопрос о том, сколько ног у животных в комнате. Вскользь упоминается, что ещё стоит и кровать. Если человек забыл про ножки кровати, он лох, если посчитал их, то тоже лох, ибо это не ноги, а ножки. Если посчитал ноги животных, то тоже лох, ибо у них лапы. Короче, вне зависимости от ответа человек – лох и ставит жирафа на аватар). А так как его действия (которые сначала нам показались такими) неправильные, то наше образование – говно и всё такое. Но если копнуть глубже, то действительно встаёт вопрос – а сколько? Если в реальной жизни в важном месте встретить такое, то, независимо от правильного ответа, нужно серьёзно поговорить с человеком, который написал это выражение и не уточнил, что он имел в виду.

Да, помню в какой-то методичке по экономике (у нас слабо вёлся этот предмет, и методички слабые были) была буквенная формула с такой же проблемой. Знак деления, справа большое достаточно выражение. Я тогда засомневался, в итоге нашёл правильную формулу. Да, там после деления всё должно было быть знаменателем. Но там это было однозначно неверно. Люди, пишите не правильно, а понятно 🙂

Правила знаков

Минус и плюс – это признаки отрицательных и положительных чисел в математике. Они по-разному взаимодействую с собой, поэтому при выполнении каких-либо действий с числами, например, деление, умножение, вычитание, сложение и т.д., необходимо учитывать правила знаков. Без этих правил вы никогда не сможете решить даже самую простую алгебраическую или геометрическую задачу. Без знания этих правил, вы не сможете изучить не только математику, но и физику, химию, биологию, и даже географию.

Рассмотрим подробней основные правила знаков.

Деление.

Если мы делим «плюс» на «минус», то получаем всегда «минус». Если мы делим «минус» на «плюс», то получаем всегда также «минус». Если мы делим «плюс» на «плюс», то получаем «плюс». Если же мы делим «минус» на «минус», то получим, как ни странно, также «плюс».

Умножение.

Если мы умножаем «минус» на «плюс», то получаем всегда «минус». Если мы умножаем «плюс» на «минус», то получаем всегда также «минус». Если мы умножаем «плюс» на «плюс», то получаем положительно число, то есть «плюс». Тоже самое касается и двух отрицательных чисел. Если мы умножаем «минус» на «минус», то получим «плюс».

Вычитание и сложение.

Они базируются уже на других принципах. Если отрицательное число будет больше по модулю, чем наше положительное, то результат, конечно же, будет отрицательный. Наверняка, вам интересно, что же такое модуль и зачем он тут вообще. Все очень просто. Модуль – это значение числа, но без знака. Например -7 и 3. По модулю -7 будет просто 7 , а 3 так и останется 3. В итоге мы видим, что 7 больше, то есть выходит, что наше отрицательное число больше. Вот и выйдет -7+3 = -4. Можно сделать еще проще. Просто на первое место ставить положительное число, и выйдет 3-7 = -4, возможно кому-то так более понятно. Вычитание действуют полностью по такому же принципу.

Правила при умножении (делении) чисел

Множители
(делимое и делитель)
Результат
+++
+
+
+

Почему нельзя делить на ноль?

«Делить на ноль нельзя!» — большинство школьников заучивает это правило наизусть, не задаваясь вопросами. Все дети знают, что такое «нельзя» и что будет, если в ответ на него спросить: «Почему?» А ведь на самом деле очень интересно и важно знать, почему же нельзя.

Всё дело в том, что четыре действия арифметики — сложение, вычитание, умножение и деление — на самом деле неравноправны. Математики признают полноценными только два из них — сложение и умножение. Эти операции и их свойства включаются в само определение понятия числа. Все остальные действия строятся тем или иным образом из этих двух.

Рассмотрим, например, вычитание. Что значит 5 – 3? Школьник ответит на это просто: надо взять пять предметов, отнять (убрать) три из них и посмотреть, сколько останется. Но вот математики смотрят на эту задачу совсем по-другому. Нет никакого вычитания, есть только сложение. Поэтому запись 5 – 3 означает такое число, которое при сложении с числом 3 даст число 5. То есть 5 – 3 — это просто сокращенная запись уравнения: x + 3 = 5. В этом уравнении нет никакого вычитания. Есть только задача — найти подходящее число.

Точно так же обстоит дело с умножением и делением. Запись 8 : 4 можно понимать как результат разделения восьми предметов по четырем равным кучкам. Но в действительности это просто сокращенная форма записи уравнения 4 · x = 8.

Вот тут-то и становится ясно, почему нельзя (а точнее невозможно) делить на ноль. Запись 5 : 0 — это сокращение от 0 · x = 5. То есть это задание найти такое число, которое при умножении на 0 даст 5. Но мы знаем, что при умножении на 0 всегда получается 0. Это неотъемлемое свойство нуля, строго говоря, часть его определения.

Такого числа, которое при умножении на 0 даст что-то кроме нуля, просто не существует. То есть наша задача не имеет решения. (Да, такое бывает, не у всякой задачи есть решение.) А значит, записи 5 : 0 не соответствует никакого конкретного числа, и она просто ничего не обозначает и потому не имеет смысла. Бессмысленность этой записи кратко выражают, говоря, что на ноль делить нельзя.

Самые внимательные читатели в этом месте непременно спросят: а можно ли ноль делить на ноль? В самом деле, ведь уравнение 0 · x = 0 благополучно решается. Например, можно взять x = 0, и тогда получаем 0 · 0 = 0. Выходит, 0 : 0=0? Но не будем спешить. Попробуем взять x = 1. Получим 0 · 1 = 0. Правильно? Значит, 0 : 0 = 1? Но ведь так можно взять любое число и получить 0 : 0 = 5, 0 : 0 = 317 и т. д.

Но если подходит любое число, то у нас нет никаких оснований остановить свой выбор на каком-то одном из них. То есть мы не можем сказать, какому числу соответствует запись 0 : 0. А раз так, то мы вынуждены признать, что эта запись тоже не имеет смысла. Выходит, что на ноль нельзя делить даже ноль. (В математическом анализе бывают случаи, когда благодаря дополнительным условиям задачи можно отдать предпочтение одному из возможных вариантов решения уравнения 0 · x = 0; в таких случаях математики говорят о «раскрытии неопределенности», но в арифметике таких случаев не встречается.)

Вот такая особенность есть у операции деления. А точнее — у операции умножения и связанного с ней числа ноль.

Ну, а самые дотошные, дочитав до этого места, могут спросить: почему так получается, что делить на ноль нельзя, а вычитать ноль можно? В некотором смысле, именно с этого вопроса и начинается настоящая математика. Ответить на него можно только познакомившись с формальными математическими определениями числовых множеств и операций над ними. Это не так уж сложно, но почему-то не изучается в школе. Зато на лекциях по математике в университете вас в первую очередь будут учить именно этому.

Ответил: Александр Сергеев

Ученые назвали правильный ответ в спорном примере из школьного курса математики — Общество

МОСКВА, 1 августа. /Корр. ТАСС Олеся Кулинчик, Александра Рыжкова/. Правильный ответ в примере из школьной математики с делением и умножением, породившем споры в социальных сетях, — «16». Об этом ТАСС заявили известные российские математики.

28 июля один из пользователей опубликовал в Twitter пример из школьной программы по математике: «8:2(2+2)=?». Обсуждение примера вызвало широкий резонанс, и перешло на международный уровень, пользователи разных стран получали ответ «16» или «1».

Российский математик, доктор физико-математических наук, первый декан факультета математики Высшей школы экономики Сергей Ландо рассказал ТАСС, что правильный ответ в России будет 16. «На территории Российской Федерации деление и умножение имеют равные приоритеты. В США или Англии может быть другой порядок. В России сначала выполняется операция в скобках, потом деление на эту сумму, а потом результат умножается на следующий множитель. Правильный ответ — 16», — сказал он. Ландо добавил, что в подобных спорных случаях специалисты стараются обозначить порядок операций скобками.

Заведующий кафедры высшей математики Национального исследовательского университета «Московский институт электронной техники» (НИУ МИЭТ) Александр Прокофьев подтвердил ТАСС, что правильный ответ — 16, и объяснил, почему пример вызвал столько споров.

«Ошибаются, как я полагаю, преимущественно взрослые. У школьников вопросов быть не должно. Первой выполняется операция в скобках, затем, согласно приоритету арифметических действий, деление и умножение — они являются равноправными и выполняются слева направо. Студенты привыкают отделять косой чертой числитель от знаменателя, поэтому путаются в данном примере, полагая, что умножение двойки на скобку расположено в знаменателе», — сказал Прокофьев.

С ними согласилась и заведующая кафедры «Математика» Российского университета транспорта Людмила Кочнева. «Если бы стояла скобка после знака деление, то правильным ответом была бы единица. Если бы после восьмерки была горизонтальная черта — знак дробного деления — а внизу 2(2+2), это была бы единица. А раз все это в строчку, вы должны делать операции в том порядке, в котором они написаны. Восемь делим на два, четыре умножаем на 2+2, получается 16. Это просто манера записи, ничего интересного — чисто арифметическая задача, но все-таки более опрятно надо писать сам пример», — пояснила она. 

Математики решили пример, вызвавший споры в соцсетях – Москва 24, 02.08.2019

Фото: портал мэра и правительства Москвы

Известные математики раскрыли правильный ответ в примере с делением и умножением. Спорная задача вызвала дискуссии в соцсетях, передает ТАСС.

28 июля в Twitter опубликовали пример из школьной программы по математике: «8:2(2+2)=?». Поиски правильного ответа вышли на международный уровень: пользователи из разных стран называли два возможных варианта – «16» или «1».

Как пояснил доктор физико-математических наук, первый декан факультета математики ВШЭ Сергей Ландо, в России правильный ответ – «16», так как в нашей стране «деление и умножение имеют равные приоритеты». При этом, например, «в США или Англии может быть другой порядок», отметил математик.

По словам Ландо, «в России сначала выполняется операция в скобках, потом деление на эту сумму, а потом результат умножается на следующий множитель». В итоге получается «16», заключил ученый.

Почему школьный пример вызвал так много споров, рассказал завкафедрой высшей математики НИУ МИЭТ Александр Прокофьев. «Ошибаются, как я полагаю, преимущественно взрослые», – отметил он, добавив, что школьники, как правило, знают, что первой выполняется операция в скобках. Потом, уточнил математик, пример нужно решать «согласно приоритету арифметических действий», а так как деление и умножение равноправны, то и действия выполняют слева направо. «Студенты привыкают отделять косой чертой числитель от знаменателя, поэтому путаются в данном примере, полагая, что умножение двойки на скобку расположено в знаменателе», – пояснил Прокофьев.

Ранее сообщалось, что столичные школьники с успехом выступили на международных олимпиадах по биологии и математике. Математическая олимпиада проходила с 10 по 22 июля в Великобритании. В ней приняли участие команды школьников из 112 стран. Им предстояло показать свои знания по геометрии, алгебре и теории чисел. В итоге золотые медали получили Тимофей Ковалев и Олег Смирнов из школы-интерната имени Колмогорова. Обладателями серебряных наград стали Валерий Кулишов и Иван Гайдай-Турлов из школы № 57.

Читайте также

Математический пример разделил Твиттер на два лагеря из-за разных вариантов ответов. При этом оба правильные Статьи редакции

8÷2(2+2) — пример выглядит так, и он составлен неверно.

{«id»:109088,»url»:»https:\/\/tjournal.ru\/internet\/109088-matematicheskiy-primer-razdelil-tvitter-na-dva-lagerya-iz-za-raznyh-variantov-otvetov-pri-etom-oba-pravilnye»,»title»:»\u041c\u0430\u0442\u0435\u043c\u0430\u0442\u0438\u0447\u0435\u0441\u043a\u0438\u0439 \u043f\u0440\u0438\u043c\u0435\u0440 \u0440\u0430\u0437\u0434\u0435\u043b\u0438\u043b \u0422\u0432\u0438\u0442\u0442\u0435\u0440 \u043d\u0430 \u0434\u0432\u0430 \u043b\u0430\u0433\u0435\u0440\u044f \u0438\u0437-\u0437\u0430 \u0440\u0430\u0437\u043d\u044b\u0445 \u0432\u0430\u0440\u0438\u0430\u043d\u0442\u043e\u0432 \u043e\u0442\u0432\u0435\u0442\u043e\u0432. \u041f\u0440\u0438 \u044d\u0442\u043e\u043c \u043e\u0431\u0430 \u043f\u0440\u0430\u0432\u0438\u043b\u044c\u043d\u044b\u0435″,»services»:{«vkontakte»:{«url»:»https:\/\/vk.com\/share.php?url=https:\/\/tjournal.ru\/internet\/109088-matematicheskiy-primer-razdelil-tvitter-na-dva-lagerya-iz-za-raznyh-variantov-otvetov-pri-etom-oba-pravilnye&title=\u041c\u0430\u0442\u0435\u043c\u0430\u0442\u0438\u0447\u0435\u0441\u043a\u0438\u0439 \u043f\u0440\u0438\u043c\u0435\u0440 \u0440\u0430\u0437\u0434\u0435\u043b\u0438\u043b \u0422\u0432\u0438\u0442\u0442\u0435\u0440 \u043d\u0430 \u0434\u0432\u0430 \u043b\u0430\u0433\u0435\u0440\u044f \u0438\u0437-\u0437\u0430 \u0440\u0430\u0437\u043d\u044b\u0445 \u0432\u0430\u0440\u0438\u0430\u043d\u0442\u043e\u0432 \u043e\u0442\u0432\u0435\u0442\u043e\u0432. \u041f\u0440\u0438 \u044d\u0442\u043e\u043c \u043e\u0431\u0430 \u043f\u0440\u0430\u0432\u0438\u043b\u044c\u043d\u044b\u0435″,»short_name»:»VK»,»title»:»\u0412\u041a\u043e\u043d\u0442\u0430\u043a\u0442\u0435″,»width»:600,»height»:450},»facebook»:{«url»:»https:\/\/www.facebook.com\/sharer\/sharer.php?u=https:\/\/tjournal.ru\/internet\/109088-matematicheskiy-primer-razdelil-tvitter-na-dva-lagerya-iz-za-raznyh-variantov-otvetov-pri-etom-oba-pravilnye»,»short_name»:»FB»,»title»:»Facebook»,»width»:600,»height»:450},»twitter»:{«url»:»https:\/\/twitter.com\/intent\/tweet?url=https:\/\/tjournal.ru\/internet\/109088-matematicheskiy-primer-razdelil-tvitter-na-dva-lagerya-iz-za-raznyh-variantov-otvetov-pri-etom-oba-pravilnye&text=\u041c\u0430\u0442\u0435\u043c\u0430\u0442\u0438\u0447\u0435\u0441\u043a\u0438\u0439 \u043f\u0440\u0438\u043c\u0435\u0440 \u0440\u0430\u0437\u0434\u0435\u043b\u0438\u043b \u0422\u0432\u0438\u0442\u0442\u0435\u0440 \u043d\u0430 \u0434\u0432\u0430 \u043b\u0430\u0433\u0435\u0440\u044f \u0438\u0437-\u0437\u0430 \u0440\u0430\u0437\u043d\u044b\u0445 \u0432\u0430\u0440\u0438\u0430\u043d\u0442\u043e\u0432 \u043e\u0442\u0432\u0435\u0442\u043e\u0432. \u041f\u0440\u0438 \u044d\u0442\u043e\u043c \u043e\u0431\u0430 \u043f\u0440\u0430\u0432\u0438\u043b\u044c\u043d\u044b\u0435″,»short_name»:»TW»,»title»:»Twitter»,»width»:600,»height»:450},»telegram»:{«url»:»tg:\/\/msg_url?url=https:\/\/tjournal.ru\/internet\/109088-matematicheskiy-primer-razdelil-tvitter-na-dva-lagerya-iz-za-raznyh-variantov-otvetov-pri-etom-oba-pravilnye&text=\u041c\u0430\u0442\u0435\u043c\u0430\u0442\u0438\u0447\u0435\u0441\u043a\u0438\u0439 \u043f\u0440\u0438\u043c\u0435\u0440 \u0440\u0430\u0437\u0434\u0435\u043b\u0438\u043b \u0422\u0432\u0438\u0442\u0442\u0435\u0440 \u043d\u0430 \u0434\u0432\u0430 \u043b\u0430\u0433\u0435\u0440\u044f \u0438\u0437-\u0437\u0430 \u0440\u0430\u0437\u043d\u044b\u0445 \u0432\u0430\u0440\u0438\u0430\u043d\u0442\u043e\u0432 \u043e\u0442\u0432\u0435\u0442\u043e\u0432. \u041f\u0440\u0438 \u044d\u0442\u043e\u043c \u043e\u0431\u0430 \u043f\u0440\u0430\u0432\u0438\u043b\u044c\u043d\u044b\u0435″,»short_name»:»TG»,»title»:»Telegram»,»width»:600,»height»:450},»odnoklassniki»:{«url»:»http:\/\/connect.ok.ru\/dk?st.cmd=WidgetSharePreview&service=odnoklassniki&st.shareUrl=https:\/\/tjournal.ru\/internet\/109088-matematicheskiy-primer-razdelil-tvitter-na-dva-lagerya-iz-za-raznyh-variantov-otvetov-pri-etom-oba-pravilnye»,»short_name»:»OK»,»title»:»\u041e\u0434\u043d\u043e\u043a\u043b\u0430\u0441\u0441\u043d\u0438\u043a\u0438″,»width»:600,»height»:450},»email»:{«url»:»mailto:?subject=\u041c\u0430\u0442\u0435\u043c\u0430\u0442\u0438\u0447\u0435\u0441\u043a\u0438\u0439 \u043f\u0440\u0438\u043c\u0435\u0440 \u0440\u0430\u0437\u0434\u0435\u043b\u0438\u043b \u0422\u0432\u0438\u0442\u0442\u0435\u0440 \u043d\u0430 \u0434\u0432\u0430 \u043b\u0430\u0433\u0435\u0440\u044f \u0438\u0437-\u0437\u0430 \u0440\u0430\u0437\u043d\u044b\u0445 \u0432\u0430\u0440\u0438\u0430\u043d\u0442\u043e\u0432 \u043e\u0442\u0432\u0435\u0442\u043e\u0432. \u041f\u0440\u0438 \u044d\u0442\u043e\u043c \u043e\u0431\u0430 \u043f\u0440\u0430\u0432\u0438\u043b\u044c\u043d\u044b\u0435&body=https:\/\/tjournal.ru\/internet\/109088-matematicheskiy-primer-razdelil-tvitter-na-dva-lagerya-iz-za-raznyh-variantov-otvetov-pri-etom-oba-pravilnye»,»short_name»:»Email»,»title»:»\u041e\u0442\u043f\u0440\u0430\u0432\u0438\u0442\u044c \u043d\u0430 \u043f\u043e\u0447\u0442\u0443″,»width»:600,»height»:450}},»isFavorited»:false}

55 648 просмотров

В Твиттере набрал популярность математический пример, который рассорил пользователей соцсети, получавших разные ответы. Однако оказалось, что правы и первые, и вторые, а проблема заключается в неправильной нотации примера.

Подписчики, посчитайте

В реплаях подписчики разделились на два лагеря — у одних получилось 16, а у других — 1. Первые сразу же принялись издеваться над вторыми и посоветовали подтянуть математику, а другие отвечали тем же. Даже калькуляторы показывали разные ответы.

Всё, кроме 1, абсолютно неверно

ВЫ МЕНЯ ДОСТАЛИ

Подождите…

i do parenthesis, multiplication, then division so i believe it’s 1

103

5830

Сначала я подсчитала в скобках, потом умножила, потом разделила, так что у меня получился 1

Получится 16, боже, реплаи просто стыдно читать…

Ответ — 1. Всем, кто говорит 16, надо пересдать математику

Очевидно, что ответ — 1

@pjmdolI it’s 16 and if you say otherwise start packing your clown shoes 🤡

3

233

Получится 16, если вы говорите что-то иное, то начинайте распаковывать свои клоунские ботинки

Ответ — 100

Некоторые из вас провалили математику, и это заметно

Никогда не видел кого-то, насколько уверенного в неправильном ответе

Никогда не видел кого-то, настолько уверенного в том, что он называет правильный ответ неправильным

Некоторые даже начали приводить свои научные степени и количество изученных курсов в качестве доказательства того, что их ответ — единственно верный.

У меня две степени по математике, ответ — 1

Я прошла 3 курса по вычислениям, дифференциальным уравнениям и линейной алгебре, получается 16, бро

Расхождение в ответах получилось из-за того, что одни пользователи сначала умножали, а другие, наоборот, делили.

В итоге пользователи Твиттера решили, что у всех получаются разные ответы в зависимости о того, какой порядок вычислений они выучили в школе. Те, кто сначала умножал, использовали метод PEMDAS со следующим порядком вычислений: сначала раскрыть скобки, а потом умножать, делить, складывать и вычитать. Те, кто сначала делил, а потом умножал, использовал BODMAS — в нём умножение и деление имеют одинаковый приоритет, а вычисления производится слева направо.

Если посчитать по PEMDAS, то ответ — 1. Если по BODMAS, то — 16

@pjmdolI In PEMDAS multiplication and division take equal priority so you do whichever is first from left to right. Same for addition and subtraction. https://t.co/1617gChUms

2

142

При PEMDAS умножение и деление имеют одинаковый приоритет, нужно посчитать то, что идёт первым слева направо. То же касается сложения и вычитания

Поскольку одна часть мира использует PEMDAS, а другая BODMAS, алгебра может иногда стать запутанной

Однако на самом деле проблема заключается в нотации — авторы примера забыли расставить скобки, поэтому правильных ответов получилось два.

И 1, и 16 — правильные ответы, потому что нотация уравнения неверна. Можете теперь наконец нахрен заткнуться?

@korchasa Вот именно, что это не вопрос приоритетов. Это вопрос нотации. Типа я пишу a/b(x + y) — это дробь a/b умножается на (x + y). Если я хочу всю правую часть в знаменатель, то a/(b(x + y)

8

Издатель «N+1» Андрей Коняев рассказал TJ, что скептически относится к подходам к порядку вычислений: «Люди — ленивые жопы. Их ломает расставлять скобки. Поэтому придумываются все эти подходы к порядку вычислений, которые, конечно, никому нах** не нужны и только всех запутывают. А потом эти порядки начинают принимать как математическую данность, хотя это просто интеллектуальный костыль».

Он также отметил, что в задачке не поставили знак умножения, что «тоже не добавило осмысленности». По словам Андрея, пример должен был выглядеть так — 8:(2*(2 + 2)). В этом случае правильный ответ равняется 1. А в этом — (8:2)*(2 + 2) — 16.

Арифметика

— можете ли вы опровергнуть это правило PEDMSA? — (деление перед умножением, вычитание перед сложением)

Чтобы расширить ответ Losethegame

Losethegame ответил: «Я попал сюда, погуглил тот же вопрос, хотя я не уверен, что (м) кто-либо из комментаторов ответил на него конкретно. Я думаю, вы правы в том, что ваше правило не может быть нарушено. Это, вероятно, может быть доказано алгебраически, потому что a * (b / c) = (a / c) * b и a + (bc) = (ac) + b …? »

Я согласен с тем, что это, вероятно, может быть доказано алгебраически с помощью методов, подобных тому, о котором упоминает losethegame (хотя пример losethegame может быть ошибочным, поскольку, как указывает user21280, losethegame меняет местами операнды).Но, развивая идею алгебраического доказательства (и без перестановки операндов!), Я могу придумать некоторую алгебру, которая могла бы это доказать. (Aand предоставил user21280 считает, что мои примеры не учитывают все возможности. Его ответ с использованием логических формул мог бы).

Дано уравнение, в котором умножение идет текстуально перед делением, например 3 * 4/2 неважно, что вы сделаете в первую очередь. Так что буквальные PEMDAS или PEDMAS подойдут. Принимая во внимание уравнение, в котором деление происходит в текстовом виде до умножения e.грамм. 6/2 * 3 , тогда имеет значение, что вы сделаете в первую очередь. Традиционное прочтение PEMDAS или PEDMAS дает это правильно (потому что они говорят, что сначала делайте первый, и это разделение), буквальное PEDMSA дает это правильно. Буквальное прочтение PEMDAS ошибочно.

Вычитание и раздел этой собственности. Допустим, мы составили небольшое уравнение той части уравнения, в которой операторы конкурируют друг с другом. Если вычитание конкурирует с сложением и вычитание происходит первым (текстуально, в уравнении), оно должно быть выполнено первым.Если деление соперничает с умножением, и деление идет первым, деление должно быть выполнено первым. В то время как если бы сложение конкурировало с вычитанием, то независимо от того, происходит ли оно первым в тексте, не имеет значения, выполняется ли сначала сложение или вычитание. Точно так же, если умножение конкурирует с делением, если умножение происходит сначала в текстовом виде, тогда не имеет значения, выполняется ли сначала деление или сначала выполняется умножение. Таким образом, литерал PEDMSA всегда работает (т.е.е. строго деление перед умножением, вычитание перед сложением). Как и традиционное / правильное чтение PEMDAS / PEDMAS, то есть чтение, в котором говорится, что умножение и деление имеют равный приоритет и сначала выполняют первое, аналогично сложению и вычитанию.

Так, например,

Принимая 1 * 2/3 , поэтому a * b / c
Следуя PEDMSA буквально дает 1 * (2/3) , поэтому a (b / c) После PEDMSA традиционно получается (1 * 2) / 3 , поэтому (ab) / c

И мы знаем алгебраически, что a (b / c) = (ab) / c

  1 * 2/3 а * б / с
знак равно
1 * 2/3 ab / c
  

Принимая 1/2 * 3 , поэтому a / b * c Это оценивается одинаково независимо от того, следует ли строго упорядоченному PEDMSA или традиционному PEMDAS i.е. Независимо от того, делаете ли вы сначала деление, как правило, или первое из умножения и деления, это (a / b) * c в обоих случаях, так что очевидно одно и то же.

  1/2 * 3 (а / б) * в
знак равно
1/2 * 3 (а / б) * в
  

С 1 + 2-3 , если мы сделаем a + b-c , который заказан PEDMSA, то есть a + (b-c) , это будет тот же результат, что и при использовании традиционного PEMDAS (a + b) -c. Мы знаем алгебраически a + (b-c) = a + b-c = (a + b) -c

Я вспоминаю, как мой учитель математики указывал на одну вещь, о которой вы хотите остерегаться / знать, — это - (a + b) , который мы просверлили в was -a-b, сильно отличается от -a + b .Вычитания всегда должны выполняться в первую очередь и по порядку … и если сначала выполняется вычитание или сначала первое из сложения и вычитания, то мы поддерживаем это правило.

  1 + 2-3 а + (б-в)
знак равно
1 + 2-3 (а + б) -с
  

И для этого то же самое. алгебраически

  1-2 + 3 (1-2) +3 (а-б) + в
знак равно
1-2 + 3 (1-2) +3 (а-б) + в
  

И я полагаю, наконец. 1-2-3 и 1/2/3 И независимо от того, выполняются ли строго заказанные PEDMSA или традиционные PEMDAS, это (1-2) -c и (1/2) / 3 так же там.

Я не уверен, все ли это возможности.

Это может оставить вопрос о том, что объясняет алгебру, например правило, что

  a * (b / c) = (ab) / c

а также

а + (b-c) = (a + b) -c
  

Также это соглашение о синтаксическом анализе, которое, кажется, разработано в конце 20-го века, а не фундаментальное правило математики. https://www.quora.com/Is-the-order-of-operations-unclear-for-expressions-like-20-2-5+5

Неявное умножение? — Доктора математики

Я хочу завершить эту серию темой, которая постоянно возникает как в классах, так и в социальных сетях: как вы оцениваете такие выражения, как \ (a \ div bc \) или \ (8 \ div 4 (3-1) \ ), где умножение указано без конкретного символа? Есть несколько причин, по которым можно интерпретировать это иначе, чем рассмотренное нами правило, согласно которому умножение и деление выполняются слева направо.Сначала мы рассмотрим это с точки зрения учащихся и учителей, а затем (в следующий раз) исследуем некоторые исторические вопросы, чтобы завершить серию.

Два способа оценить ax ÷ по

Давайте сначала рассмотрим один из предыдущих вопросов, которые мы задавали по этой проблеме в 1999 году, чтобы подготовить почву:

 Порядок действий

Проблема была представлена ​​так:

   а = 1,56
   b = 1,2
   х = 7,2
   у = 0,2

   ax / by =?

Я решил эту проблему двумя способами:

1) Я сначала переписал задачу как [1.56 (7,2) / 1,2] (0,2). Во-вторых, a было умножено на x. Продукт был 11,232. Затем, поскольку скобок не было, я следил за порядком операций и разделил 11,232 на b, что составило 1,2. Частное было 9,36.  Затем я умножил 9,36 на y, что составило 0,2. Окончательный ответ - 1,872.

2) По-другому, первое, что я сделал, это умножил a на x. Продукт, который был равен 11.232, был временно отложен. Затем b был умножен на y, что дало произведение 0,24. Теперь проблема была решена разделением 11.232 (или топор) на 0,24 (или на), чтобы получить окончательный ответ 46,8.

Подскажите, пожалуйста, какой ответ правильный и почему? 

(Обратите внимание, что в то время единственным способом ввести разделение в нашем электронном письме было использование косой черты \ (a / b \), которая, как я обычно предполагаю, представляет собой выражение, фактически записанное как \ (a \ div b \). Время от времени я буду вставлять обелус, ÷, где мы предприняли грубые попытки имитировать его.)

Первый способ следует за PEMDAS буквально, как обычно учат и как я представил его здесь, вычисляя слева направо как \ (a \ cdot x \ div b \ cdot y = ((a \ cdot x) \ div b ) \ cdot y \).

Второй видит это как \ (ax \ div by = (ax) \ div (by) \). Это не объясняется как следование какому-либо изучаемому правилу, а как выполнение того, что выглядит правильно, либо потому, что деление читается, как если бы это была дробная черта, либо просто потому, что « на » выглядит так, как будто оно принадлежит друг другу как единое целое. Мы увидим несколько причин, по которым студенты сделали это.

Хотя я проработал с Ask Dr. Math менее года, это был уже знакомый вопрос, на который я хотел подробно ответить ради архива:

 Не только вы задаетесь этим вопросом.У нас было несколько других вопросов о выражениях, похожих на ваш, от сбитых с толку учителей и учеников, которые обнаружили, что разные книги или учителя дают разные ответы, и даже калькуляторы расходятся. 

Обратите внимание, что не только студенты делают то, что считают правильным, но и некоторые учебники и калькуляторы используют второй метод.

Новое правило, или что выглядит правильным?

Я подробно остановился на двух методах, приняв версию PEMDAS за правильную (хотя у меня есть несколько дополнительных соображений по этому поводу):

 Как написано, ваше выражение

    топор / by

должно быть вычислено  слева направо : умножить на x, разделить на b, умножить на y. Умножение не выполняется перед делением, но оба выполняются в том порядке, в котором они появляются. Ваше первое решение правильное.

  Некоторые тексты содержат правило , как и во втором решении , что умножение без символа («подразумеваемое умножение») должно выполняться перед любыми другими операциями в выражении  [кроме экспонент], включая «явное умножение» с использованием символа. . Следуя этому правилу, вы должны умножить a на x, затем умножить b и y, а затем разделить одно на другое.В некоторых (вероятно, в большинстве) текстах такое правило не упоминается, но  некоторые из них могут использовать его, не говоря об этом, что намного хуже . 

Мне кажется, что я придумал термин « подразумеваемое или неявное умножение », когда несколько месяцев назад отвечал на свой первый вопрос по этой теме, для обозначения умножения, обозначенного простым помещением двух чисел или переменных или выражений в скобках рядом с друг друга — « сопоставление », как другие называют это — например, \ (ab \), \ (2b \) или \ (a (b + c) \), в отличие от явного написания \ (a \ times b \ ) или \ (a \ cdot b \).

Мы видели несколько вопросов от студентов, чьи учебники преподавали только обычный PEMDAS, но оценивали второй способ на примерах или решениях без комментариев. Это могло произойти из-за того, что ответы на обратной стороне были написаны кем-то, кроме автора, но это непростительное несоответствие.

Зачем автору вводить это дополнительное правило? В разное время у меня были разные мнения о том, является ли правило хорошей идеей, но я всегда признавал, что это не то, чему обычно учат:

 Я не знаю общего правила среди математиков, согласно которому подразумеваемое умножение должно выполняться перед явным умножением.Насколько мне известно, все умножения умещаются в одном и том же месте в порядке операций.  Это не безосновательное правило , однако, поскольку кажется, что подразумеваемое умножение связывает операнды вместе более плотно, , по крайней мере, визуально ; но идея порядка операций (или приоритета, как его называют в компьютерном мире) должна гарантировать, что все будут интерпретировать неоднозначное выражение одинаково - так что , если некоторые тексты изменяют правила, или если люди делать то, что кажется естественным, цель потеряна . 

Правило, которое не является правилом, бесполезно, каким бы разумным оно ни было. Да, «новое правило» — это естественный способ чтения \ (ax \ div by \), потому что \ (by \) выглядит как единое целое; но пока все этому не научат, мы не можем ожидать, что нас поймут все читатели.

В частности, многие студенты предполагают, что он представляет собой горизонтальную версию \ (\ displaystyle \ frac {ax} {by} \):

 Проблема в том, что выражение выглядит так, как будто оно должно быть

     топор
    ----
     от

В Dr.Часто задаваемые вопросы по математике относительно написания математики в электронной почте, одна из наших рекомендаций -  использовать круглые скобки везде, где это возможно, чтобы избежать двусмысленности , даже если правила должны прояснять это, потому что в некоторых ситуациях их можно легко забыть.

Итак, в электронном письме мы бы написали это так:

    ax / (by) или (ax / b) * y

в зависимости от того, что задумано. 

Используя круглые скобки, мы можем избежать написания того, что люди, которых учили другим правилам или которые игнорируют правила, которым их учили, могли бы воспринять иначе, чем мы.

Выпуски калькулятора

 При исследовании другого «пациента» доктора математики я обнаружил, что некоторые калькуляторы экспериментировали с этим правилом. У калькуляторов несколько иные потребности, чем у математиков, поскольку они должны принимать ввод линейно, один символ за другим, поэтому они вынуждены принимать решение по этому поводу. На веб-сайте TI я узнал, что они намеренно добавили эту «функцию» в TI 82, а затем убрали ее из TI 83, вероятно потому, что они решили, что это не стандартное правило и может сбить с толку людей.

Ссылка там давно испортилась; но когда в 2008 году возник конкретный вопрос о калькуляторе, я процитировал то, что TI сказала в своей базе знаний:

 Калькуляторы предполагаемого умножения и TI

...
Решение 11773: предполагаемое умножение против явного умножения на графических калькуляторах TI. 

Имеют ли подразумеваемое умножение и явное умножение одинаковый приоритет на графических калькуляторах TI?

  Подразумеваемое умножение имеет более высокий приоритет, чем явное умножение , что позволяет пользователям вводить выражения таким же образом, как они были бы написаны.Например, TI-80, TI-81, TI-82 и TI-85 оценивают 1 / 2X как 1 / (2 * X), ​​в то время как другие продукты могут оценивать то же выражение как 1/2 * X слева направо. верно. Без этой функции было бы необходимо сгруппировать 2X в круглых скобках, что обычно не делается при написании выражения на бумаге.

Этот порядок приоритета был изменен для семейства TI-83, семейства TI-84 Plus, семейства TI-89, TI-92 Plus, Voyage ™ 200 и карманного компьютера TI-Nspire ™ в режиме TI-84 Plus. Подразумеваемое и явное умножение имеют одинаковый приоритет.

Это проясняет, что разработчики калькуляторов должны выбирать свои собственные правила, которые не должны совпадать с правилами написания на бумаге; но педагоги, кажется, убедили их сохранить как можно больше ради учеников.

В заключение (вернемся к ответу 1999 года):

 Итак, чтобы ответить на ваш вопрос, я думаю, что  оба ответа можно считать правильными  - что, конечно же, означает, что  сам вопрос неверен . Я предпочитаю стандартный способ (ваш первый ответ) при разговоре со студентами, , если только их собственный текст не дает правило  «сначала неявное умножение»; но на практике, если бы я натолкнулся на это выражение, я бы, вероятно, сначала проверил, откуда оно взялось, чтобы понять, что было задумано.Главный урок, который нужно усвоить, заключается не в том, какому правилу следовать, а в том, как избежать двусмысленности в том, что вы пишете сами.  Не доставляйте другим людям такие неприятности.  

Впоследствии у нас появилось еще много вопросов по этому поводу; Я просто процитирую несколько уникальных моментов из некоторых из этих ответов.

Старомодная математика?

Вот типичный пример школьного конфликта 2000 года:

 Порядок ведения спора

Задача гласит: N ÷ ml, где n = 12, m = 6 и l = 3. Я считаю, что правильный ответ должен быть.6666, так как 12, разделенное на 18, равняется этому. Мой муж со мной согласен.

Мой сын пришел домой из школы очень расстроенным, с запиской своего учителя, что ответ был неправильным. Она указала, что мне следовало разделить 6 (m) на 12 (n), прежде чем я разделил 3 (l) на уравнение. Ее ответ был 6.

Мой сын очень расстроен мной; его учитель сказал ему, что я занимаюсь «старомодной математикой». Мне нужно вернуться в школу? 

Задача \ (N \ div ml \), и родители производят умножение первыми.Я ответил частично:

 Я могу сообщить вам хорошие и плохие новости. Во-первых, плохие новости: в соответствии с обычным порядком действий, которым теперь обучают, ваш ответ неверен. ... 

Я объяснил стандартные правила и добавил:

 НО ...

Вы не одиноки в своем мнении. Эта часть правила - одновременное умножение и деление - вероятно, последнее правило, которое стабилизировалось; Я знаю, что, по крайней мере, в 20-е годы соглашения не было. Кажется, что соглашение было достигнуто, но сейчас оно распадается, как я слышу от многих студентов, чьи тексты отвечают на подобные вопросы так же, как и вы.Похоже, что они добавляют  неустановленное правило, которое кажется вполне разумным в данном контексте , что подразумеваемое умножение (обозначенное простым помещением двух переменных или выражений вместе, как в "ml") должно быть выполнено первым. Конечно,  выглядит как , как будто это должно означать это. Проблема в том, что, хотя я слышал, что это правило  следует за  часто, я почти никогда не слышал о том, что  преподает , поэтому эти тексты не следуют своим собственным установленным правилам.

В следующий раз я расскажу подробнее об истории.

 Поскольку этот тип выражения настолько неоднозначен, что люди не соглашаются с правилами, а правила легко игнорировать, мое собственное мнение таково, что  ни ваш ответ, ни ответ учителя не верны: вопрос неверен .  Ни один ответственный математик не стал бы писать такое выражение; мы бы просто сказали

     п
    ---
    м л

так что не было бы никаких сомнений в его значении. В конце концов, цель правил - позволить нам ясно общаться, а не помочь нам обмануть учеников и начать ссоры между семьями.Так что вы на самом деле можете быть «старомодным»; или вы можете быть на передовой. В любом случае, боюсь, вам просто нужно будет узнать, как они это делают в классе, и следить за ними. Больше не о чем беспокоиться. 

В последнее время драки, как правило, происходят в социальных сетях!

Неправильное применение распределительного свойства

Я закончу самым последним заархивированным обсуждением. Это вопрос от 2017 года:

 Еще больше о порядке операций

Мне любопытно узнать, каков ответ на это:

   8/4 (3–1)

Если строго следовать PEMDAS, ответ - 4:

   8/4 (2)
   2 * 2
   4

Однако, если вы следуете распределительному свойству, вы получите 1:

   8 / ((4 * 3) - (4 * 1))
   8 / (12 - 4)
   8/8
   1

Какой из них будет правильным и почему?
   
Оба действительны, поэтому я не согласен с тем, какой ответ был бы правильным.Он должен быть правильным или неправильным, а не двумя разными ответами. 

Я ответил собранием моих стандартных ответов на такого рода вопросы; даже мой первый заархивированный ответ на эту тему в 1999 году был в основном стандартным ответом, который я давал другим раньше. Здесь я просто рассмотрю несколько замечаний, которые не были полностью рассмотрены выше.

Я сначала резюмировал, что происходило:

 Проблема не в конфликте между PEMDAS и дистрибутивом; это то, что  строгая интерпретация  PEMDAS конфликтует с  естественным впечатлением  о значении выражения, так что вы неосознанно применяете альтернативную интерпретацию, когда думаете, что просто применяете свойство распределения.

Если вы помните более ранние утверждения о том, что PEMDAS (а) соответствует свойствам операций и (б) соответствует визуальному впечатлению от нашей нотации, то некоторые тревожные звонки уже должны звучать!

 Когда вы распределяли, вы ПРЕДПОЛАГАЛИ, что именно 4, а не 8/4 умножали (3 - 1).  2 \ div 4b + c \): 

 На самом деле, есть несколько разных причин, по которым люди приводят (это очень популярный вопрос), некоторые из которых лучше других.Как утверждает ваш друг, правила, которым обычно учат, говорят нам делать все умножения и деления слева направо (в пределах любого их кластера) и не делать исключений, которые заставили бы сначала вычислить 4b. Многие из нас здесь согласятся с этим, и покончить с этим.

Некоторые люди сначала оценили бы 4b из-за неправильного понимания PEMDAS, , думая, что это означает, что умножение должно выполняться до деления . Я думаю, вы знаете, что они ошибаются.

Еще одна неправильная причина, применяемая к несколько иному типу выражения, - это неправильное понимание  скобок : правило, согласно которому скобки «предшествуют» всему остальному, заставляет их поверить, что в выражении вроде 12/4 (4-1) умножение 4 (4-1) должно быть выполнено в первую очередь.Но правило круглых скобок на самом деле говорит только о том, что сначала нужно оценить то, что ВНУТРИ круглых скобок; результат обрабатывается как любое другое число. (Иногда я называю это представлением « залипающих скобок ».)

Другой причиной, приведенной в отношении этого второго типа выражения, является идея о том, что распределительное свойство   вынуждает вас сначала выполнить умножение, потому что они сначала вычисляют 4 (4-1) = 4 * 4-4 * 1 = 12, а затем делить; но здесь возникает вопрос, потому что единственная причина, по которой они взяли 4, а не 12/4, в качестве множителя слева, заключается в том, что они так им казались.И, конечно же, свойство распределения - это всего лишь способ, которым вы можете, если хотите, переписать выражение, чтобы получить то же значение; это вне вопроса о том, что само по себе означает выражение.

В конечном счете, большинство людей, вероятно, делают это просто , потому что это кажется правильным : 4b выглядит ближе друг к другу, поэтому мы, естественно, хотим сделать это в первую очередь.  Но они не могут указать ни одного правила, которое бы это оправдало; а поскольку математика - это доказательство, и то, что вы ЗНАЕТЕ, правильно, а не только то, что кажется правильным, это плохо.

Пример «круглых скобок» см. В

.
 Связана ли двойка с числами в скобках? 

Пример того, как знак деления виден как дробная черта (и долгое обсуждение того, что внешний вид не влияет на него), см. В

 Порядок операций и дробей 

Назад к ответу 2017 года…

Избегание - лучшая политика

 В книгах и рукописной математике, выходящей за рамки элементарного уровня, мы почти никогда не используем символ горизонтального деления, а вместо этого используем дробные черты, что не оставляет двусмысленности.В результате у математического сообщества никогда не было необходимости делать выбор в этой ситуации!  По сути, его оставили неопределенным , и именно авторы учебников придумали явные «правила» для описания того, что на самом деле является языком, который развился органически, , основанный не на тщательно сформулированных правилах, а на молчаливом соглашении .

Итак, какой «правильный» способ читать такое выражение, зависит от того, какие правила действуют в конкретном сообществе (математический класс, журнал и т. Д.) - и что было задумано автором.

Я закончил с призывом к миру:

 В результате в таких проблемах, как эта, ошибка в первую очередь совершается не теми, кто дает «неправильные» ответы, а теми, кто изначально публикует проблему (или передает ее). Любой, кто действительно хочет правильно выполнять математические вычисления, захочет четко рассказать об этом и будет избегать всего двусмысленного или неопределенного. Они должны быть либо полностью заключены в круглые скобки, либо использовать горизонтальную дробную полосу, чтобы упорядочить порядок:

      6 6
   -------- или --- (2 + 1)
   2 (2 + 1) 2 

Споры в социальных сетях по этому поводу - пустая трата времени. Но размышления о наших условностях могут быть очень полезными. В следующий раз я закрою все, взглянув на историю и приведу несколько веских причин считать «новое правило» правильным.

Неоднозначно PEMDAS

14.04.2014: ссылок
  • Пример форума hpmuseum
  • Пример физического форума: 48/2 (9 + 3)
  • защитников Делите и умножайте ранжируйте поровну и идите слева направо. и есть другие мемноники, такие как «Ешьте, пожалуйста, вкусные яблочные штрудели мамы».
  • В этом научном блоге упоминается статья Тара Хэлле что довольно хорошо уже говорит о том, что происходит (если бы я видел эту статью, написанную 12 марта 2013 г., Я бы не стал записывать это, потому что в этой статье очень четко говорится, что первоначальная оценка того, что нет договоренность о порядке умножения или деления верна). Тем не менее, эта тема побудила меня сказать что-то новое о порядке операций одного и того же типа, например, D или E в PEMDAS, что выходит за рамки споров о BEDMAS.Вот интересная цитата из той статьи
          "Интернет-слухи утверждают, что Американское математическое общество написало" умножение, указанное сопоставлением, осуществляется
          перед делением ", но в сети больше не существует оригинального источника AMS (если он вообще существовал). Тем не менее, некоторые ранние учебники по математике
          также учил студентов делать все умножения, а затем все деления, но большинство из них, например, эта алгебра средней школы 1907 года
          учебник, этот учебник 1910 года и этот учебник 1912 года рекомендовали выполнять все умножения и деления в
          порядок, в котором они появляются первыми, затем следуют сложения и вычитания.(Это соглашение имеет смысл также и с канадским
          и британские версии PEMDAS, такие как BEDMAS, BIDMAS и BODMAS, которые все перечисляют деление перед умножением на
          (аббревиатура). Самый разумный совет, содержащийся в «Mathematical Gazette» за 1917 год, рекомендовал использовать круглые скобки для
          избегать двусмысленности.  (Да!) Но даже известный историк математики Флориан Каджори написал в «Истории математических обозначений»
          в 1928-1929 гг .: «Если арифметический или алгебраический член содержит / и х, в настоящее время нет согласия относительно того, какой
          знак должен использоваться первым."
           
    В статье есть ссылки на источники учебников. Вот запись 242 в книге Флориана Каджориса «История математической записи» (стр. 274), которая упоминается в этой цитате. Я не вижу никаких указаний на рекомендации, данные в цитируемых учебниках для старших классов, например здесь, здесь, но упомянутая запись в книге Вебстера Уэллса об этом ясно сказано на странице 18:

Обновление от 18 мая 2017 г .: В последнее время загадки вроде ниже появились, которые упускают из виду, что количество картофеля фри изменилось или что используется одна вишня) всплыли Эти головоломки успели стать вирусными не из-за PEMDAS, а потому что люди не смотрят на варианты (3 вместо 4 банана, 2 часа, а не 3 часа).Поначалу почти все ошибаются. Но есть еще и проблема PEMDAS. Некоторые получают 88. Но чтобы получить 88, нужно было написать скобка (2 + 3 + 3) * 11. (спасибо Абите Сукумаран за то, что поделился этим).
Обновление от 2 августа 2017 г .: Преш Талвалкар пишет
«Я делаю математические видео на YouTube на канале« MindYourDecisions ».
Некоторые из самых популярных видео - это неоднозначные выражения, связанные с порядком действий.
В ходе исследования я наткнулся на ваш веб-сайт и обнаружил проблему:
Что такое 2x / 3y - 1, если x = 9 и y = 2?
Я бы ответил 11, как сказал учитель 5-го класса.Я был ошеломлен тем, что ни один из 60 студентов вашего гарвардского курса математики
ответил 11 (вы объяснили, что 58 получил ответ 2; а затем 2 получил ответ 18/5).
 
Мой ответ:
«да, это интересная вещь. Конечно, ни один из ответов не« правильный »
поскольку мы знаем, что интерпретации BEDMAS и PEMDAS могут
использоваться без нарушения каких-либо полномочий.  Как указано на странице, ответ
11 - это то, что есть у большинства компьютерных языков. Вас явно этому учили.
Было бы интересно узнать, какой процент людей говорит 11.Мои эксперименты говорят, что это очень
редкий. Большинство делают умножение перед делением, так как PEMDAS кажется более
популярны и больше преподают в школах. БЕДМЫ PE (MD) AS, кажется, преподают
значительно меньше. Единственное, что мы знаем, это то, что утверждение, что одним из ответов является
единственный правильный ответ - это неправильно ".
 

Обновление от 5 августа 2017 г .: Джейкоб Пошоланн Кефоед Кристенсен присылает другой пример и замечание по поводу обелуса.
"Проблема в том, что изображение мобильного телефона получает 9 из уравнения: 6 ÷ 2 (2 + 1)
что, по их мнению, будет 1.В своем споре вы определяете obelus и разделительную косую черту как имеющие разные значения.
Ну да, на самом деле они имеют два разных значения, и поэтому обычно
никогда не используйте обелус. Только американец может по-прежнему использовать его, но этот знак был удален
в использовании уравнений научных работ из-за его исторической проблемности.
Первое слово obelus в Северной Европе означает вычитание.
Во-вторых, obelus рекомендуется удалить в научном использовании, потому что у нас уже есть знак для любого из них (разделительная косая черта («/») и вычитание («-»)).Хотя, по вашему мнению, обелус и косая черта деления должны означать два
разные значения У вас часто есть только одна опция на калькуляторе, чтобы сделать знак деления ".
 
Мой ответ: 
«Спасибо за пример 6 ÷ 2 (2 + 1). Он тоже показывает неоднозначность. Да, в зависимости от того, кто входит в команду PEMDAS или PEDMAS, получает 1 или 9. Это тоже прекрасный пример, когда можно увидеть жаркие дискуссии. Как указывалось и ранее другими в список литературы, там нет правильного ответа .3 / (3 + у). Я был удивлен и должен был написать на доске пояснение: Экзамен по-прежнему прошел хорошо. На этой фотографии, сделанной незадолго до экзамена, вы можете увидеть, что все были счастливы уйти: Урок очень ясен: как учитель, даже если вы знаете лучше, вы должны быть очень четкий, даже избыточный. Даже если нет двусмысленности, лучше быть на всякий случай.
Кстати, статья в Википедии упоминает пример
1 + 2x3 = 9 Калькулятор Microsoft в стандартном виде
1 + 2x3 = 7 Калькулятор Microsoft в представлении программистов
 
Он показывает, что один и тот же поставщик в рамках, где нет двусмысленности (никто никогда не сомневается, что умножение должно предшествовать сложению), двусмысленность в том же товар.В другом примере из этой статьи упоминаются калькуляторы Texas Instruments.
1 / 2x = 1 / (2x) в калькуляторе TI-82
1 / 2x = (1/2) x в калькуляторе TI-83
 
Самопровозглашенные правила вроде это вряд ли поможет.
Обновление от 19 января 2018 г .: Тимоти Масгроув любезно обратил мое внимание на глупую дискуссию о youtube в котором вопрос of 6 ÷ 2 (1 + 2) снова появляется (см. выше). Также эта история показывает, насколько богословские дебаты может стать уже тем фактом, что часть зрителей, которым нравится видео и неприязнь к видео примерно одинакова, показывает, что ответ на эту проблему должен быть неоднозначным.Выше я привел (частично вслед за Тарой Хэлле, которая написала этот Slate article), исторические указатели, показывающие, насколько неоднозначны вещи. Вот лагеря:
  1. PEMDAS (умножение предшествует делению)
  2. PEDMAS (деление предшествует умножению)
  3. PE (MD) AS (Деление и умножение имеют одинаковый вес, зависит от того, что осталось)
  4. Неоднозначно (Нет установленного правила)
Компьютеры в основном следуют за вторым или третьим.Большинство людей и особенно студенты (экспериментально) склонны следовать правилу PEMDAS. Литература указывает на неоднозначность.
PEMDAS BEDMAS PE (MD) AS
6/2 * (1 + 2) 1 9 9 (1 + 2) * 6/2 9 9 9
Есть причина, по которой лагерь "PE (MD) AS" чувствует себя намного лучше. У нас в обоих случаях одинаковые отвечать. Также компьютеры часто следуют «PE (MD) AS» и придерживаются точки зрения «слева» на «право».
Еще хуже, вероятно, спорят, когда спрашивают, что такое 8 ÷ 2/2 (некоторая средняя школа Учитель подтвердил мне, что деления (знаки обелуса и обратной косой черты) в некоторых учебниках трактуются по-разному, см. замечание "obelus" выше, сделанное Якобом Пошоланом Кефоедом Кристенсеном. Некоторые скажут, что ответ - 8, потому что / стоит перед ÷. Если двигаться слева направо, получаем 2.

Обновление от 4 сентября 2018 г. :

Я получил следующее приятное письмо:

Как, черт возьми, можно сказать, что это двусмысленно, когда это АКСИОМАТИЧЕСКОЕ, что умножение и деление являются обратными операциями? Как можно сказать, что это неоднозначно, когда ЛЮБОЕ деление может быть выражено как умножение на обратное? Позор вам за увековечивание ерунды.

Единственное, что немного беспокоит, так как писатель на самом деле кажется учителем. Независимо от аргумента, писатель, вероятно, должен перейти в профессию, где требуется как можно меньше человеческого взаимодействия. Я ответил

Уважаемый ...,

, вы, вероятно, ссылаетесь на http://www.math.harvard.edu/~knill/pedagogy/ambiguity/

Дело не в том, является ли деление обратным умножению. Это определение.2/3. Теперь, если вы посмотрите на литературу и историю, то оказывается, что нет однозначного ответа, что правильно. И если это так, мы назовем это неоднозначным.

Есть лагерь, который защищает PE (MD) AS, где MD равны и где порядок имеет значение, если умножение используется вместе. Но это только усложняет ситуацию, поскольку у нас есть три разных интерпретации.

Итак, если кто-то пишет такое выражение, как x / 3x, он должен быть осторожно и поставил кронштейны.Все остальное может вызвать недопонимание.

Вы не единственный, кто чувствует себя очень сильным и эмоционально из-за этого.


Обновление от 2 октября 2018 г. :

Мне прислали ссылку на следующий адрес YouTube видео. На данный момент это один из лучших материалов на YouTube. Хорошо видно, что в реальном мире выражения используется по-другому: например, в опубликованных статьях mn / rs обычно в публикациях интерпретируется как (mn) / (rs) или лекций Фейнмана, можно увидеть, что 1 / 2N 1/2 интерпретируется как 1 / (2 N 1/2 ).В инженерии можно прочитать W = PVMg / RT. Еще один замечательный момент, сделанный в этом видео, заключается в том, что можно написать x / 2 если 1 / 2x интерпретируется как (1/2) x. Никто бы не написал 1 / 2x, если бы они означает x / 2. Итак, на практике интерпретируется выражение как 1 / (2x), которое является PEMDAS, но отличается от BEDMAS или интерпретация того, что умножение и деление лежат в одном и том же опора. Также упоминается, что в руководстве AMS есть PEMDAS (умножение предшествует делению). Также следует руководство Американского физического общества. ПЕМДАС.Видео еще раз демонстрирует, что единственный способ избежать двусмысленность заключается в использовании скобок.


24 октября 2018 г. : Изначально я планировал опубликовать на YouTube версию несколько слайдов от 28 апреля 2018 г. Harvard Extension STEM Club, но на это не было времени. Спасибо Ana Carolina Smith за возможность выступить. Вот часть слайдов:
СЛАЙДЫ PDF (76 стр.)
26 октября 2018 г. : Другой хороший пример от кого-то: Вот письмо:
Мне сказали, что когда вы умножаете и делите
(так как порядок работы значения не имеет)
вам никогда не нужно использовать круглые скобки, верно?
Потому что 2 * 3/4 ​​* 6 на моем калькуляторе дают мне 9,
и я ожидал 0.25! Для меня это должно быть
эквивалентно 2 * 3 / (4 * 6), потому что, поскольку мы не
нужны круглые скобки, это единственный способ набрать его без них.
Если я хочу вычислить 2 * 3/4 ​​* 6, как мой калькулятор
да, я должен набрать 2 * 3 * 6/4, это правильно?
 
Мой ответ:
Порядок операций имеет значение.  Вам нужно поставить
скобка. Мне нравится ваш пример. Это
уже хорошо это иллюстрирует. Большинство людей получат
6/24 = 1/4, как и вы. Большинство языков программирования
(компьютеры) дают 9. Компьютер следует PEDMAS
(деление перед умножением)

  2 (3/4) 6 = 9

или используйте правило (MD), которое означает «все, что будет первым»

 ((2 * 3) / 4) * 6 = 9

Люди (и большинство рекомендаций, таких как профессиональные
такие общества, как AMS, следуют PEMDAS, что означает
вы сначала делаете умножение, а затем деление

 (2 * 3) / (4 * 6) = 1/4

Но следовать рекомендации не имеет смысла
если существуют разные интерпретации и компьютеры это делают
разные.3)) = 7625597484987

компьютер идет справа налево.
Также здесь необходимы скобки.
 
4 ноября 2018 г. : С.А. добавил в историю интересный ракурс: Рекомендуется сначала упростить, а затем удалить скобки.
Я читал ваш блог по вопросам программирования на MD или DM.
Проблема в том, что все они противоречат первому закону алгебры.
Упростите, а затем УДАЛИТЬ круглые скобки.
Все эти соглашения нарушают это, говоря только упрощать скобки ВНУТРИ.

Итак, сначала мне нравится, что вы сказали AMBIGUOUS на 6/2 (1 + 2)
1 или 9

Однако я вздремнул, астрально переместился к старому Евклиду, и он засмеялся.Доказательство 1 и 9.

6 / x = 1 или 6 / x = 9
Когда x = 2 (1 + 2)

2 (1 + 2) = 2 (3) = 6

6/6 = 1

Таким образом, не учить студентов убирать скобки в новой математике, это противоречит первому закону алгебры.
Все эти условные обозначения аббревиатур необходимо исправить, чтобы они соответствовали 1-му закону алгебры.
Итак, согласны ли вы, что новые математические соглашения должны согласовываться с первым правилом алгебры Евклида?
Думаю, да.
 
Вот мой ответ:

Это интересный ракурс. Но учтите, что рекомендация  "упростить" 
Здесь находится проблема неоднозначности: 
Да, можно упростить 6/2 (1 + 2), введя x = 2 (1 + 2) = 6, а затем получить 6/6 = 1 Но можно также упростить, определив x = 6/2, а затем получить x (1 + 2) = 9. Собственно, это тоже исторически интересно. Вы упомянули Евклида. Евклид не использовал известную нам алгебру. Символическая алгебра появилась только с Вите в 16 веке. Насколько нам известно, только в ХХ веке реализовали что здесь действительно есть двусмысленность. Об этом ясно сказано в книге Каджори о математическая нотация, которая является авторитетом в этом вопросе.
Это тоже стало педагогическим вопросом: студентов сегодня в основном обучают правилу PEMDAS, которое формально ставит умножение перед делением и рекомендовал бы результат 6/2 (1 + 2) = 1.Если дать выражение системе компьютерной алгебры, они все дают 6/2 (1 + 2) = 9. Все эти обсуждения были вызваны такими примерами. Первое правило алгебры по-прежнему остается хорошим правилом. Это хороший совет. к несчастью это не устраняет двусмысленность. Но я согласен, что это помогает писателю избегать двусмысленность. Но знаете, в основном проблема возникла в образовательных учреждениях. Если учитель спрашивает ученика, что такое 6/2 (1 + 2), учитель не хочет упростите это, так как это уже решит проблему.Если сегодня учитель спросит студенты, что такое 6/2 (1 + 2), то это просто напрашивается на неприятности. Правильно do - это уточнить и написать либо (6/2) (1 + 2), либо 6 / (2 (1 + 2)). Каджори уже было ясно, что отказ от скобок не дает четко определенные математические выражения. Оливер
3 декабря 2018 г. : Atmos добавил еще один интересный ракурс
Потенциальным решением этого противоречия может быть то, что когда у вас есть
коэффициенты и переменные, записанные вместе без операторов между
тема.грамм. 5ab, мы можем рассматривать это как вложенную операцию.
Другими словами, отсутствие символа оператора означает, что оператор
отношения между ними имеют приоритет над любыми внешними операциями,
т.е. 5ab представляет собой (5 * a * b).

Итак, если у вас был / bc, записан только один оператор (разделение
символ), а часть "bc" будет подразумеваться вложенной из-за
упущение оператора внутри. Так что это все равно будет "a over bc",
как именно это выглядит и сколько из нас учили.А потом, если
нам нужно указать, что операция между a и b фактически занимает
приоритет над отношениями между b и c, тогда мы просто
вместо этого напишите a / b * c. Ни суеты, ни суеты.

Разве это не более эффективный способ общения с
математический язык здесь? И разве не в этом суть математического
язык, чтобы эффективно передавать концепции? В противном случае этот вид
путаница никогда не исчезнет, ​​и нам придется написать намного больше
круглые скобки в наших уравнениях (и никто не хочет этого делать).Немного
людям нравится "новая математика" сверхстрогой интерпретации PEMDAS
особенно потому, что это простой способ обмануть людей и сделать математику
более запутанно, чем должно быть. Тем не менее, это, кажется, побеждает все
Дело в том, почему мы вообще это делаем.

У меня есть оба способа сделать это, но строгий метод PEMDAS кажется
контрпродуктивен, потому что он вызывает так много проблем и делает вещи
например, превратить якобы простую дробь вроде 2x / 3y в фактическ
имея в виду 2xy / 3 вместо этого, что кажется совершенно безумным.Но если вместо этого
мы просто используем PEMDAS, когда операторы на самом деле написаны, тогда все
такого рода проблемы исчезнут буквально в одночасье. «Старая математика»
и "новая математика", наконец, согласится, и мы сможем все это сделать
с одним очень простым правилом.

Что ты об этом думаешь?
 
Я ответил
Привет, Атмос,
отказ от знаков умножения уже обычно делается.
На самом деле большую часть времени. Однако может возникнуть дополнительная проблема.
при использовании чисел, а не переменных вроде 3/45 не то же самое, что 3/4 5
Но вы вносите интересный момент, потому что теперь их стало еще больше.
двусмысленность:

             3/45 = 3 больше 45 = 1/15
             3 / (4 * 5) = 3 больше 20 = 3/20
             (3/4) 5 = 3/4 умножить на 5 = 15/4

Проблема PEMDAS - это не «проблема, которую нужно решать».Это вопрос
Дело в том, что существуют разные интерпретации и что человек для
пример считывает x / yz с x = 3, y = 4 и z = 5 как 3/20, в то время как машина
(практически все языки программирования) дают другой результат.
Есть авторитеты, которые установили правила (большинство учеников учат
PEMDAS), что является одной из причин, по которой многие люди спрашивали о 3/4 * 5, давая 3/20
которые большинство машин просили дать 15/4:
 
Я набираю это в Mathematica
       х = 3; у = 4; z = 5; x / y z и получаем 15/4

Это лингвистическая проблема, а не математическая.В случае
лингвистическая проблема, ее нельзя решить путем введения нового правила.
Единственный способ решить проблему - избежать ее. Можно избежать этого, чтобы
поставить скобки.

Оливер
 
Обновление от 14 декабря 2018 г. :

В новейшем гайде по мультфильмам из серии Ларри Гонника (которые фантастические), есть еще кое-что о Порядок операций. Но далеко не идёт. "Если без скобок присутствуют, умножьте и разделите перед сложением и вычитанием ". Это очень грубое правило, но оно имеет то преимущество, что он не попадает в войны PEMDAS.


Обновление от 18 января 2019 г. :

Учитель математики прислал мне следующий пример. Здесь не только неоднозначность PEMDAS. Также вопрос "96 разделить на 6 из 4" появляется, что может означать "96/6 умножить на 4" или затем «96 / (6 * 4)». Это особенно интересный случай, потому что того, что:

Вопрос:
Я учитель математики и недавно столкнулся с конкретным вопросом
на PEMDAS (пожалуйста, проверьте приложение), где ученики получили два разных ответа (6 и 66).Причиной получения двух разных ответов было то, как студенты решили последнюю часть вопроса:
96 ÷ 6 из 4 
Метод 1: Некоторые ученики решили это следующим образом: 96 ÷ 24
Метод 2: Остальные решили это как: 16 x 4
Мне нужна ваша помощь, чтобы определить, какой метод правильный, или оба являются правильными. методы приемлемые.
Мой ответ: в этой проблеме есть две двусмысленности, и да, все ответы даны. студентами должны быть оценены как правильные.

1) Первое выражение: похоже, что ученики истолковали 57 ÷ 19 * 2 выражение равно 6, даже если это может быть 3/2, если используется PEMDAS (и официальные рекомендации AMS или физического общества и используется в большинстве научных работ, особенно если выражения являются переменными). Что происходит, так это то, что если бы вопрос был задан как 57/19 * 2, то многие интерпретировали бы его. как 57/38.
2) Третье выражение - новая вещь, поскольку «of» как «умножение» необычно.Также здесь нет определенных правил. Оба ответа 6 и 66 в целом верны.
Я бы даже посчитал 3/2 правильным, так как это то, что получается, если использовать правило PEMDAS и не правила PEDMAS или PE (MD) AS. Итак, вот четыре возможных ответа. Слева мы теперь всегда есть однозначные выражения:

57 / (19 * 2) -64 * 2/32 + 96 / (6 * 4) = 3/2
(57/19) * 2-64 * 2/32 + 96 / (6 * 4) = 6
(57/19) * 2-64 * 2/32 + (96/6) * 4 = 66
57 / (19 * 2) -64 * 2/32 + (96/6) * 4 = 123/2
 
Пример снова показывает, что скобки нужно ставить всегда.Но это также показывает, что может произойти, если для описания арифметических операций используется "разговорный язык", так как это может может привести к другим двусмысленностям. «Что составляет две трети от 9» должно быть ясно как (2/3) * 9, а 2 ÷ 3 из 9 тоже можно интерпретировать как 2 / (3 * 9). Этот пример снова указывает на то, что люди могут интерпретировать знак обелуса ÷ иначе, чем знак деления /.
Обновление от 2 мая 2019 г. :
В швейцарской газете 20 Min задача 6/2 (1 + 2) = ??? тоже упоминается.К статье уже добавлено 1384 комментария. Как и в течение многих лет в социальных сетях, борьба продолжается там. Самое интересное, насколько большинство уверены в своей правоте со всех сторон. Что снова указывает на двусмысленность.

Название статьи: «Миллионы не справляются с этим математическим уравнением!» В качестве «доказательства» есть видео на YouTube, которое дает ответ 9. Автор этого видео, Преш Талуокер дает в своем блог ссылка Леннес, Н. Дж. "Дискуссии: Относительно порядка операций в алгебре."The American Mathematical Monthly 24.2 (1917): 93-95 ..." Лучше прочитать эту статью.
В этой статье 1917 года действительно утверждается, что «большинство учебников» используют правило слева направо, если деление и умножение кажутся смешанными. Но в нем также указано «установленное правило»

«Все умножения должны быть выполнены в первую очередь, а затем деления».

Итак, у нас есть это: это просто нонсенс, что 12 миллионов человек, которые делают это по-другому, не были «неспособны решить проблему».Мы определенно имеем дело с ситуацией, которую следует считать неоднозначной. Статья 1917 года - хорошая ссылка. Это уже подтверждает. Но с 1917 г. Правилу PEMDAS научили миллионы людей. Поразительно только то, как многие утверждают, что знают правильный ответ. Может быть, это просто человеческая природа.

Прочтите в конце статью Леннеса, который писал уже в 1917 году:

"Когда способ выражения получил широкое распространение, его нельзя изменить по желанию.Это дело лексикографа и грамматик записывать, а не то, что, по его мнению, должно означать выражение но что на самом деле понимают те, кто его использует. Язык алгебры содержит определенные идиомы, и при формулировании грамматики языка мы должны обратите внимание на них. Например, 9a 2 ÷ 3a означает 3a и not 3a 3 - такая идиома. Дело не логическое, а историческое.

Лучше не скажешь! Значит, идиоты не 12 миллионов человек.Те, кто так утверждает, есть.

Обновление от 5 августа 2019 г. : Стивен Строгац уступает а New York Times пишет новый поворот и обвиняет несоответствие между оценкой и средняя школа: взят пример 8 ÷ 2 (2 + 2), где каждый компьютер дает 16, а люди обычно дают ответ 1. В статье снова очень четко говорится, что вопрос неоднозначный. Нет правильного или неправильного, если есть разные противоречивые правила. Единственные, кто утверждает, что существует одно правило, ошибаются!
Это досадное математическое уравнение? Вот дополнение. Путаница (скорее всего
намеренно) сводится к несоответствию используемых математических правил
в начальной и средней школе.8 ÷ 2 (2 + 2) =?

Проблема заключалась в том, что он дал два разных ответа, 16 или 1,
в зависимости от порядка, в котором выполнялись математические операции.
выполненный. В юном возрасте студенты-математики обучаются особым навыкам.
соглашение о «порядке операций», которое диктует порядок следующим образом:
круглые скобки, показатели, умножение и деление (подлежат рассмотрению
на равных, с разрывом галстуков, работая слева направо), и
сложение и вычитание (также равного приоритета, со связями аналогично
сломанный).Я утверждал, что строгое соблюдение этой элементарной конвенции PEMDAS:
приводит только к одному ответу: 16.

Тем не менее, многие читатели (включая моего редактора), одинаково приверженные тому, что
они рассматривали в качестве стандартного порядка операций, усиленно настаивал
правильный ответ был 1. Что происходило? После прочтения
много комментариев к статье, я понял, что большинство из этих респондентов были
используя другое (и более сложное) соглашение, чем элементарное
Конвенция PEMDAS, которую я описал в статье.В этом более сложном соглашении, которое часто используется в
алгебры, неявному умножению дается более высокий приоритет, чем явному
умножение или явное деление, в котором записываются эти операции
явно с такими символами, как x * / или ÷. Под этим более изощренным
соглашение, неявное умножение на 2 (2 + 2) дано выше
приоритет, чем явное деление на 8 ÷ 2 (2 + 2). Другими словами,
2 (2 + 2) следует оценить в первую очередь. Это дает 8 ÷ 2 (2 + 2) = 8 ÷ 8 =
1. По тому же правилу многие комментаторы утверждали, что выражение 8 ÷ 2 (4)
не было синонимом 8 ÷ 2x4, потому что круглые скобки требовали немедленного
разрешение, что снова дает 8 ÷ 8 = 1.Это соглашение очень разумно, и я согласен, что ответ - 1.
если мы будем его придерживаться. Но это не принято повсеместно. Калькуляторы
встроенные в Google и WolframAlpha используют более элементарное соглашение;
они не делают различия между явным и неявным умножением
при указании вычислить простые арифметические выражения.

Подпишитесь на Science Times Мы расскажем вам истории, отражающие
чудеса человеческого тела, природы и космоса.
Более того, после того, как Google и WolframAlpha оценивают все, что находится внутри
набор круглых скобок, они эффективно удаляют круглые скобки и не
больше расставляйте приоритеты по содержанию.В частности, интерпретируют 8 ÷ 2 (2 + 2)
как 8 ÷ 2x (2 + 2) = 8 ÷ 2x (4), и обрабатываем это как синоним 8 ÷ 2x4. Затем,
согласно элементарному PEMDAS, деление и умножение имеют
равный приоритет, поэтому работаем слева направо и получаем 8 ÷ 2x4 = 4x4
и получили ответ 16. В своей статье я решил сосредоточиться на этом
более простое соглашение.

Другие комментаторы возражали против самого исходного вопроса. Посмотри как
они отметили, что это было плохо поставлено. Это можно было бы сделать намного яснее
если бы в нужном месте был вставлен только другой набор круглых скобок,
записав его как (8 ÷ 2) (2 + 2) или 8 ÷ (2 (2 + 2)).Верно, но это упускает из виду: вопрос не был задан
ничего ясно. Напротив, его безвестность кажется почти
умышленно. Это, безусловно, искусно извращенное, будто построенное для
причинить шалость.

В выражении 8 ÷ 2 (2 + 2) используются круглые скобки - обычно это инструмент для сокращения
путаница - в манере джиу-джитсу, чтобы усилить мутность. Оно делает
это путем сопоставления цифры 2 и выражения (2 + 2), что означает
неявно, что они предназначены для умножения, но не помещая
явный знак умножения между ними.Зритель остается в недоумении
следует ли использовать сложное соглашение для неявного умножения
из класса алгебры или вернуться к элементарному соглашению PEMDAS
из средней школы.

Выбирает: "Итак, проблема в том, как она поставлена, смешивает обозначения начальной школы.
с обозначениями средней школы, что не имеет смысла. Человек который
хорошо помните математику в начальной школе, скажите, что ответ - 16. Люди
кто помнит свою алгебру, с большей вероятностью ответит 1. "

Как бы мы ни предпочли четкий ответ на этот вопрос,
не один.Вы говорите помидор, я говорю томахто. Некоторые электронные таблицы и программное обеспечение
системы категорически отказываются отвечать на этот вопрос - они упираются в его искаженную
состав. Это тоже мой инстинкт, как и большинство математиков, которых я
говорил с. Если вы хотите получить более четкий ответ, задайте более четкий вопрос.
 
5 августа 2019 г. . Только что появилась еще одна сокровищница Дженни Горхэм на ютубе: 8 августа 2019 г. . Грег Макканн любезно указал эта ссылка на заархивированная копия руководящих принципов AMS.{-1} dt $.

17 августа 2019 г. Другой вопрос:
Мне любопытно, какие, по вашему мнению, ответы на это уравнение.
8 ÷ 2 (4). Для меня самый простой порядок действий - это умножение 2 * 4
во-первых, потому что мне еще нужно разобраться со скобками.
Я просто считаю, что нам нужно провести какую-то математику, чтобы избавиться от скобок.
Некоторые люди просто бросают их, не делая никаких вычислений.
Я имею в виду, зачем они вообще, если их можно просто уронить в любой момент
без каких-либо математических вычислений, чтобы их очистить.С.
-------------------------------------------------- ---------------------------------

Мой ответ:

Да, это одна из последних загадок PEMDAS. Подобный
Также Строгац обсуждал в New York Times. Вы упомянули, что
2 + 2 уже оценивается, но это не меняет ситуацию на
8 ÷ 2 (2 + 2), который теперь раздается. Но это
та же история. Причина, по которой ставится скобка вокруг 4, заключается в том, что она не читается
как 24. Но это не проясняет двусмысленность.Да, хочется сначала сделать 2 * 4 и
получить результат 1. Большинство компьютерных программ оценивают его как 16. Пример:

Ядро Mathematica 12.0.0 для Linux x86 (64-бит)
Авторские права 1988-2019 Wolfram Research, Inc.
В [1]: = 8/2 (4)
Из [1] = 16

Почти все люди оценили бы его как 1.
Мнения здесь не имеют значения, поскольку теперь хорошо задокументировано, что там
просто нет консенсуса (ни авторитетом, ни историческим ростом,
это лингвистический феномен, о котором так поздно осознали необходимость его точного определения).Вещи можно интерпретировать по-разному, и так останется.
Чтобы прояснить ситуацию, необходимо разместить скобки.
 
17 сентября 2019 г. . Ответ на некоторые вопросы проверки фактов из Нью-Йорк Таймс. Я ответил:
Резюме:
-------------------------------------------------- ------------------------------------------
На вопрос 8/2 (2 + 2) есть разные ответы в зависимости от используемого правила.
Его можно интерпретировать как (8 / (2 (2 + 2))) = 1 или (8/2) (2 + 2) = 16 в зависимости от
правило.Общепринятого правила не существует, их несколько: PEMDAS, BEDMAS, PE (MD) AS.
Невозможно сказать, что правильно, а что нет. Есть
разные правила, приводящие к разным результатам. Выражение не очень хорошо определено.
Похоже, что большинство людей естественно дают ответ 1 и большинство
компьютеры и языки программирования возвращают ответ 16.
Чтобы выражение было однозначным, нужно поставить скобки.
Лишь сравнительно поздно (около 100 лет назад) стало ясно, что существует двусмысленность.Нет
С тех пор был достигнут консенсус, так что нет альтернативы для уточнения выражения.
Литература по этому поводу - Флориан Каджори, «История математической записи», Лондон, 1928 год.
Н. Дж. Леннес, Относительно порядка операций в алгебре, Amer. Математика. Ежемесячно, 24 1917
-------------------------------------------------- ----------------------------------------------

Вот ответы на ваши вопросы:

- Точно сказать, что согласно PEMDAS, «2 + 2» должны быть односторонне
 первая операция выполнена?

 Да! Но это не PEMDAS.Операция 2 + 2 выполняется первой, потому что
 вокруг него были установлены скобки.

- Согласно математике нижнего уровня, решение этого уравнения должно быть 16?

 Нет. Существует правило PEMDAS, которое широко распространено.
 (дети выучивают «Пожалуйста, извините мою дорогую тетю Салли»), и при их следовании спрашивает
 делать умножение перед делением. Это дает 8 / (2 (2 + 2)) = 1.
 Как и сегодня, у большинства детей младшего уровня математики есть доступ к калькуляторам или
 онлайн-инструментов, они могут дать ответ 16. Причина утверждения, что это
 часть математики нижнего уровня, вероятно, то, что большинство учителей теперь используют калькулятор для проверки
 вещи, и компьютер сообщает им, что ответ - 16.Как студенты и их
 родители получают разные результаты (если они не используют компьютер) есть разногласия.

- Справедливо сказать, что в алгебре или высшей математике операция 2 (4) имеет приоритет?

 Нет. Также здесь это зависит от используемого правила. Если использовать PEMDAS по назначению, то
 ответ - 1, что означает сначала вычислить 2 (2 + 2), поскольку M стоит перед D.
 Вопрос не в том, какой уровень или предмет использовать. Ответ зависит от того, какой
 правило используется.

- Согласно высшим математическим стандартам, решение этого уравнения 1?

 Нет.Также это зависит от используемого правила. При выполнении теста со студентами большинство дает
 ответ 1. На самом деле, большинство людей дают ответ 1, если они не используют компьютер.
 Большинство людей читают такие выражения, как 1. Однако компьютеры, которые рассматривают умножение
 и деление часто находится на одном уровне и почти всегда дает ответ 16.
 В 2014 году я спросил у поступающих первокурсников (еще не старших математиков) во вступлении
 курс исчисления и все, кроме одного, использовали правило PEMDAS. Большинство людей отвечает 1.

- В статье утверждается, что способ написания этого математического выражения вводит в заблуждение.Вы согласны с этим утверждением?

 Нет. Это не вводит в заблуждение, это неоднозначно. Было бы заблуждением, если бы
 правильный ответ, и выражение приведет к неправильному ответу. Это не тот случай.
 Нет правильного или неправильного ответа. Выражение неоднозначно и зависит от
 правило, которое используется.

- Справедливо сказать, что по мере того, как человек переходит на более высокий уровень математики (уровень после начальной школы),
 деление вообще обозначается как дробь?

 Нет. В высшей математике используются все виды выражений.Разделение на математику более низкого уровня
 и математика более высокого уровня менее важна (по моему опыту)
 чем вычислить, как человек читает математику естественно (и большинство учебников это делают)
 или если вычислить выражение на компьютере. Это может быть психологическое, это может быть
 быть лингвистом, это может быть благодаря тому, что его так учат, это может быть чтение текстов, но большинство людей
 умножение перед делением в ситуациях, подобных рассмотренной.

- Как бы записать это уравнение в дробной форме?

 В недвусмысленных выражениях используются скобки типа 8 / (2 (2 + 2)).
 Другая возможность - использовать выражение

   8
----------------
   2 (2 + 2)

 Это дает 1.При этом важно, чтобы была четкая
 длинная линия деления дроби, убедившись, что 2 (2 + 2)
 сделано до разделения. Другая версия была бы
 (8/2) (2 + 2), что дает ответ 16, как и большинство компьютеров.
 
2 ноября 2019 г.: На этом сайте есть онлайн-калькулятор упоминание BODMAS BODMAS также является аббревиатурой от Bracket, Order / Of, Деление, умножение, сложение и вычитание. Веб-сайт и калькулятор полностью упускают из виду суть определения выражений. что можно понимать по-разному.Тем более, что сайт называется Калькулятор PEMDAS предполагает использование правила PEMDAS, в котором умножение перед сложением. Таким образом, калькулятор оценивает 2 * 9/3 * 2-1 как 11, как и большинство калькуляторов или программистов. языки делают, но это отличается от того, что на самом деле предлагает PEMDAS (2 * 9) / (3 * 2) - 1 = 2 и которые большинство людей оценивают, когда их спрашивают. Итак, калькулятор, а не помогает чтобы прояснить присутствующую двусмысленность, это просто калькулятор. Есть современные калькуляторы, которые предупреждают учащегося о том, что выражение неоднозначно и заключено в круглые скобки, поэтому что пользователь может, если эти круглые скобки не соответствуют тому, что имелось в виду, может их изменить.

6 апреля 2020 г. Другой вопрос:

Надеюсь, что все в порядке, я пишу тебе по электронной почте. Я наткнулся на вашу страницу во время
дебаты по проблеме Facebook. Я в отпуске, так что у меня много
времени в моих руках. Эта версия была 6/2 (1 + 2). Мне ответ на
выше - 1 и только 1. Но я не хотел обсуждать это. Я хотел добавить
что-то, чего я не видел на вашем сайте, что я думаю,
больше промахов PEMDAS. Математику часто называют универсальным языком. Это
позволяет общаться людям из разных культур.Когда вы читаете
первый пример 2x / 3y - 1, когда x = 9 и y = 2, как вы это читаете? Ты
прочитать два раза x разделить на три раза y минус один? Ты читаешь это
два x больше трех y минус один? Подобно тому, как в английском языке есть правила,
в математике тоже есть правила. Английский не идеален и не всегда следует
правила. Математика такая же. Иногда умножение и группирование могут
подразумевается.
 
Мой ответ:
да, математика - универсальный язык, позволяющий общаться между
разные культуры.Но языки также были созданы людьми и
не всегда идеальны. Двусмысленность PEMDAS на самом деле просто упущение
дизайна. Не было авторитета, который раз и навсегда сказал бы, что это
это необходимо указать. Причина исторически ясна. Только один
осознал проблему слишком поздно. Вы говорите, что для вас 6/2 (1 + 2) равно 1, да,
почти все люди предполагают это. Если вы отдадите его компьютеру,
это дает вам 9, почти все языки программирования. Да вроде английский,
В математике есть правила. Но они далеки от совершенства.Языки меняются и
со временем совершенствоваться. В ходе обсуждения PEMDAS выяснилось, что
слишком поздно принимать правила. Некоторые пробовали, и многие из них фанатичны
и думают, что их путь правильный. Например, в случае 6/2 (1 + 2)
мы видим сторонников (например, на YouTube), которые спорят,
только 9 - правильный ответ. Но, как вы говорите, читать это как 6/2 (1 + 2) - это
укоренившиеся и изменяющиеся, что многих расстроило бы. Последние несколько слайдов в
эта презентация [PDF] немного показывает, что говорят лингвисты.
На самом деле это не математическая проблема, это лингвистическая проблема.То, что хорошо
об обсуждении заключается в том, что теперь все учителя и ученики знают о
двусмысленность и запишите уточненные выражения.
 

9 апреля 2020 г. Из другого электронного письма:

Я не математик. STEM всегда был моей сильной стороной, но
Я предпочитаю применение теории, поэтому я техник по оборудованию
в полупроводниковой промышленности (я чиню роботов, которые делают компьютер
фишки). Я упоминаю об этом, чтобы сказать, что я, возможно, не эксперт, но и не
мирянин.Мой первый опыт с этим вопросом возник, когда я пытался
для программирования квадратной формулы в моем калькуляторе TI-83 + почти 20
много лет назад. Чтобы уравнение работало правильно, дополнительные скобки
необходимы. Это потому, что стандартная алгебра обозначена неявным
пути, а компьютеры никогда не улавливают подразумеваемых значений или только начинают
совсем недавно. Об этом говорится в цитируемой вами статье NYT, но это
высказанное в некотором смысле я считаю ошибочным. В нем говорится "просто или элементарно"
математика 8/2 (4) такая же, как 8/2 * 4, и только в алгебре 8/2 (4) становится
8 / (2 * 4).Калькуляторы подвержены ошибкам, как вы указали MS Calculator.
решает 1 + 2 * 3 как 9, а не 7, что никогда не бывает правильным. Распределительный закон
умножения и деления »доказывает понятие двух правильных ответов
недействительным. 8/2 (2 + 2) = 16 - полное нарушение закона распределения,
таким образом недействителен. Если уравнение записано (8/2) * (2 + 2), то порядок
операции будут диктовать, что деление происходит первым, а затем результат
будет распределен во вторых скобках, как если бы он
были написаны 8/2 * (2 + 2).Однако в любом уравнении, где написано
8/2 (2 + 2), то неявно (не явно) закон распределения
должен применяться как часть круглой скобки до того, как произойдет разделение,
в явном виде это будет 8 / (2 * (2 + 2)). Дело в том, что компьютеры
(или калькуляторы) не понимают, что мы подразумеваем, не означает
значение неверно, это означает, что компьютеры могут понимать только явные
инструкции. Явное решение уравнения с подразумеваемыми факторами
обычно приводит к неправильному ответу.2-4ac) / 2a, но оба дают совершенно неверные
ответы с калькулятора. Очевидно, что квадратичная формула - это константа
и не должны получать разные результаты, независимо от того, чему вас учили,
это доказывает, что мы должны быть достаточно умными, чтобы правильно пользоваться нашим калькулятором,
не то, чтобы калькуляторы безошибочны.
Другой пример - уравнение «ab * cd». Решение этого выражения
явно бы вы сделали «a * b * c * d = x», когда мы знаем, что это на самом деле
(a * b) * (c * d), потому что () подразумеваются, как указано выше. Ваш алгебраический
пример использования 2x / 3y-1 глуп.3
как указал Леннес в своей статье 1917 года. Причина 2x / 3y-1 = 11
на калькуляторе, потому что калькулятор не понимает, что
неявная группировка. Каждый родитель понимал неявную группировку, что
(2x) / (3y) -1 - подразумеваемое уравнение, хотя явно не записано как
такой. Только когда вы введете в калькулятор 2 * 9/3 * 2-1, вы получите 11. Но
2 * 9/3 * 2-1 будет записано 2 * x / 3 * y-1, что сильно отличается от 2x / 3y-1,
но калькулятор считает их одинаковыми. Это было доказано, когда у вас
ваш класс по математике сделает это, двое из ваших учеников вбили это в свои
калькуляторы, остальные 58 подсчитали правильно.Что это значит - это ты
нужно научить этих двух учеников правильно пользоваться своими калькуляторами,
не то чтобы есть некоторая универсальная двусмысленность. При программировании четырехугольной формы
вы должны явно заключить в скобки всю неявную группировку, иначе это не сработает,
это ограничение программного обеспечения, а не недостаток математики. Если здесь
была ли какая-либо фактическая двусмысленность в результатах вашего класса исчисления
были разделены ближе к 50 на 50, статистически очевидно, что
устойчивый консенсус среди студентов, изучающих математику, относительно правильного метода.В
Учитель математики, получивший 2x / 3y-1 = 11, должен работать в другой области.
 
Мой ответ
спасибо за вашу заметку. Да, это очень интересная тема, особенно
в связи с компьютерами. Вы правы, что с калькуляторами один
нужно быть еще осторожнее и поставить больше скоб. Один из
Причины, по которым калькуляторы HP имели большой успех, заключаются в том, что они использовали
обозначение обратной полировки, позволяющее пропустить многие скобки. Я никогда не был
в том лагере HP, но, как и вы, использовали калькуляторы TI.Пример 2x / 3y
-1 не так уж и глупо. На самом деле это очень интересно. Да, вы
правильно, что каждый человек читает это как (2x) / (3y) - 1, но компьютер дает
что-то другое. Я рассчитываю это здесь с помощью Mathematica, одного из
самые продвинутые системы компьютерной алгебры, и это дает -1+ 2xy / 3 не -1+
(2x) / (3y) (см. Прикрепленный снимок экрана). Я также согласен с тем, что PEMDAS
двусмысленность не является недостатком математики, как ее часто представляют, это
просто некоторые выражения нуждаются в большей ясности (значение
скобки), чтобы иметь смысл.Немного компенсирует то, что там
вокруг так много людей, которые верят, что есть определенный путь и только
их путь правильный. В примере 2x / 3y -1 большинство людей просто естественно
предположим, что это означает (2x) / (3y) - 1, но, по мнению некоторых фанатиков,
есть только один способ увидеть это правильно, и это то, что дают компьютеры
тебе нравится -1+ 2xy / 3. Также удивительно, как долго длится это обсуждение.
продолжается. Но от этого становится еще интереснее. Нет
только математическая или лингвистическая сторона, есть также социальный аспект
к рассказу.И, как вы упомянули как инженер, это может иметь решающее значение. Если
кто-то пишет программу, управляющую роботом, и упускает из виду что-то подобное,
это просто не работает. Я рано понял, что программирование
окончательный тест понимания. Это сложнее, чем читать или писать
или учите предмет. Если процедура не работает, это доказательство того, что есть
это то, чего еще не понимаешь.
 
После другого вопроса о распределении, ведущем от главный пункт:
Я пытался подчеркнуть, что
в неоднозначной ситуации нет правильного или неправильного
нравиться

8/2 (2 + 2)

Есть ответы 1 или 16, в зависимости от того, какое правило вы
использовать.Это причина, по которой люди продолжают спорить
об этом. Ответ 1 - это ответ, который получает большинство людей.
Ответ 16 - это то, что получают большинство компьютеров.

Я пытался подчеркнуть это с самого начала в
http://www.math.harvard.edu/~knill/pedagogy/ambiguity

В своем последнем письме я указал, что это не проблема.
с распределением. 2 (2 + 2) всегда равно 8, есть
нет споров. Опять же: вопрос в том, является ли (8/2)
сначала вычисляется, а затем умножается на (2 + 2), чтобы получить
16 или сначала вычисляется 2 (2 + 2) = 8 и 8/8 = 1
получается в результате.Это вопрос о том,
деление или умножение выполняется в первую очередь. Это
собственно говоря (наблюдение, глядя, какие люди
напишите в сети), что есть те, кто верит одному
из ответов правильный.

Здесь нет правильного или неправильного. Вопрос тоже не в
вводящие в заблуждение. Есть двусмысленность. Напрашивается вывод, что
нужно более четко записывать, используя скобки.
 
И еще одно электронное письмо:
Думаю, я мог прорваться. Я принял это на данный момент
умножение через сопоставление преподавалось как высший порядок
умножения или равно всем порядкам умножения.Таким образом, 5 / 2x
учили означать (5/2) * x или 5 / (2x) в зависимости от вашего учителя.
Таким образом, единственное решение - четкое правило, определяющее умножение через
сопоставление. Как определить, является ли умножение путем сопоставления
это высший порядок или нет? Я думаю (хотя мне и больно) этот язык
и грамматика может держать ключ.
Но сначала я должен спросить о x * 2xy? Я никогда не видел, чтобы кто-нибудь писал
что-то вроде этого, как x2xy или 2xyx или что-то еще, было бы это
правильно сделать так? Если нет, то почему? Когда я вижу что-то подобное со мной
показывает, что группировка подразумевает скобки, а не просто умножение, мысль
на результат они бы не повлияли.
на что я ответил
Некоторую двусмысленность можно избежать путем сопоставления, некоторые могут быть
делается по заказу, но остается неоднозначным, особенно если набирать.
Похоже, вы все еще думаете, что двусмысленность PEMDAS - открытая проблема.
который необходимо решить. Это не открытая проблема. Было решено
100 лет назад, когда осознал, что нужно просто написать больше скобок в
Общее. Это была лингвистическая оплошность. При написании грамматики
правил для арифметики, сначала не понимали, что были
детали, требующие большего количества скоб, чем предполагалось.При использовании только умножения проблемы нет. В
причина в том, что существует ассоциативность и коммутативность продукта
операция. Уже деление не ассоциативное и не коммутативное

   (3/3) / 3 = 1/3 не равно 3 / (3/3) = 3
    3/5 не 5/3
 

13 июля 2020 г. : Учитель средней школы указал мне на статью Переосмысление порядка операций в Журнал «Учитель математики» с октября 2017 года.2 или выражения типа 2x / 3y-1, если x = 9 и y = 2. Кроме того, приведенная выше статья «Переосмысление порядка операций» делает вещи более сбивает с толку. Это другое мнение. Нет необходимости в переосмыслении, если человек остается ясным. Указывая на то, что порядок умножения, деления или возведения в степень должен быть уточнял письменно умнейший больше. Каждый студент должен знать того факта, что такие выражения, как 18/3 * 2, часто оцениваются людьми как 3 (века математических писаний закрепили это), в то время как компьютеры оценивают это как 12.2 оценивается компьютерами справа налево. Также GEMA не дает любые направляющие линии здесь. Он просто остается неоднозначным без дополнительных разъяснение. Никакого "переосмысления" не исправить. Просто используйте скобки и все проблемы решены.

4 августа 2020 г. :

Только что дочитали на вашей веб-странице о неоднозначных уравнениях.
Каково же мое удивление, узнав, что математическая конвенция не лечит
умножение и деление одинаково. Я изучал математику уже 3+
лет (прежде чем прийти в себя) и преподавал на уровне колледжа.(1 / n), x ÷ n = x * (1 / n) и x - n = x + (-n). Однажды они поймали
после этого они были поражены тем ловушкам, которых они избегали.

Несмотря на это, раскрывая давние соглашения, интересно ...
неоднозначность разрешается так, чтобы решение уравнения 8 ÷ 2 (2 + 2)
можно найти?

Я должен сказать, что я горячо утверждал, что ответ был
16, хотя я проигнорировал двусмысленность. Прочитав вашу веб-страницу,
двусмысленность очевидна. Однако мой брат, инженер на пенсии,
Ханиуэлл, чье понимание математики превосходит даже мое собственное, также пришло
вверх с ответом 16.Интересно, что его первый инстинктивный ответ был 1.

Однако ваша ссылка на ISO 31-0 и ссылка на видео YouTube
(«ПЕМДАС ошибается») отправил меня в кроличью нору. Между прочим, я
обнаружил, что ISO 80000-1 теперь заменяет ISO 31-0, fyi. К несчастью,
ISO 80000-1 недоступен бесплатно, поэтому мне пришлось прибегнуть к ISO 31-0.
для уточнения.

Возможное (?) Решение этого неоднозначного уравнения требует знания
правила и условности Я подозреваю, что большинство людей не знают.
В частности ... что обелус (÷) нарушает ISO 31-0.-1 "
(альтернативно «a * 1 / b»). (товар записывается как "а б",
«ab», «a * b» или «a x b». ), что при применении правил AMS и
APS, умножение предшествует делению. (что "/ xy" условно
математики понимают как "/ (xy)" _

Решение 1: 8 ÷ 2 (2 + 2) = 8/2 (2 + 2) (в соответствии с ISO 31-0) 8/2 (4) (на
порядок операций) 8/8 (в соответствии с правилами AMS и APS, которые умножают
предшествует делению) 1

Решение 2: 8/2 (2 + 2) можно переписать как 8 / [2 (2 + 2)] [на математическое
соглашение о том, что "/ xy" = "/ (xy)", а также из-за сопоставления, посредством чего
«2 (2 + 2)» можно было бы увидеть как «2y», используя замену, и «8 / 2y» = «8 / (2y)»]
8 / [2 (4)] 8/8 1

Многие люди заявляют, что проблема связана с лингвистикой и / или грамматикой.я
рассматривают это как проблему перевода и что это входит в компетенцию
математик, чтобы перевести задачу на общий язык. это
аналогично тому, как иммигрант впервые слышит разговорный язык.
Это напоминает мне русского комика Якова Смирнова, который немного
где он говорил о странных идиомах. Он слышал, как кто-то сказал, что бросили курить
холодная индейка. Он сказал: "Что ты куришь сейчас, ветчина?"

Итак, я теперь верю, что математик, применяющий международную
стандарты, а также соглашения AMS, могут переводить
уравнение и разрешить двусмысленность.Что вы думаете? Моя логика верна?

-------------------------------------------------- -----------------

Мой ответ:


Ваш пример - типичный пример, когда люди и машины получают
разные вещи. Если скармливать в компьютер, получается 16

[knill @ knill11:] математика
Ядро Mathematica 12.1.1 для Linux x86 (64-бит)
Авторские права 1988-2020 Wolfram Research, Inc.
В [1]: = 8/2 (2 + 2)
Из [1] = 16

Я думаю, что большинство людей получают 1 из-за обучения, PEMDAS и потому, что по математике
и физическая литература, есть неявное предположение, что 2 (2 + 2)
вместе сначала особенно с обелусом.Двусмысленность не может быть решена
путем поиска правил, руководств AMS или стандартов ISO (особенно если
они не доступны в открытом доступе) это факт (и вы подтверждаете
что), что мы можем получить разные ответы. Многие люди в Интернете пытаются
рассуждают так или иначе. Это бесполезно. Чем это поможет, если 99 из
люди рассуждают так, а 99 компьютеров дают ответы наоборот?
Да, и обелус даже немного запутывает, как вы указываете. Там
нет логического способа решить двусмысленность.Это определение неоднозначно.
Нет четкого стандарта, и если он будет, то он еще не принят.
повсеместно. Единственный способ решить проблему - поставить скобки.
 

8 августа 2020 г. : Вот проницательный комментарий Элиаса Мартенсона из Швеция:

Я читал вашу статью о порядке операций, или как вы это называете
«правило PEMDAS».

Одна вещь, которая, если упомянуть, не сразу очевидна, - это то, что когда
вы говорите, что «правилу пемдаса научили миллионы студентов»,
вы конкретно имеете в виду студентов в США.Меня учили математике в Швеции, и там нас учили, что есть
три уровня приоритета:

Возведение в степень
Умножение и деление
Сложение и вычитание

Скобки на самом деле не упоминаются, потому что
здесь нет двусмысленности. Кроме того, чтобы объяснить, что их можно использовать
, чтобы переопределить правила по умолчанию.

Сейчас я живу в Сингапуре, и мои дети ходят здесь в местную школу. я просто
проверил с ними, и они действительно узнали так же, как я
учили.

На всех языках программирования, которые я знаю, а также на математических
Программное обеспечение, которое я тестировал, отмечу, что они следуют именно этому правилу.Правило "PEMDAS" кажется способом превратить что-то довольно тривиальное в
что-то сложное. В дополнение к этому, он также преподает
это неправильно, так как если кто-то настаивает на его использовании, ему придется написать
что-то вроде: PE [M / D] [A / S], чтобы уточнить эквивалентность в
группы. Простое прочтение этого документа создает у студента впечатление, что
умножение должно как-то выполняться перед делением во всех случаях,
что редко бывает правдой.

Я говорю «редко» здесь, потому что я согласен с вами в том, что, безусловно, есть
случаях имеет смысл сначала выполнить умножение, например, ваш
пример 1 / ху.Тем не менее, если вы напишете это как 1 / x y, это не так ясно
anymore, что означает, что пробелы значительны.

Как инженер-программист, который в свободное время изучает физику], я
часто раздражают нечеткие обозначения в математике. Не только проблемы
с горизонтальным написанием (как тема вашей статьи), но также правильным
набирать уравнения с помощью LaTeX. Авторы склонны использовать сокращенные обозначения,
повторно использовать символы и т. д., что означает, что вы должны иметь в виду контекст, когда
выясняя, что происходит.Это тип проблемы, с которой я бы столкнулся
надеялись, были ограничены областью лингвистики, а не математики,
единственная научная область, где что-то можно доказать с уверенностью.

Наконец, я хотел бы добавить, что причина, по которой я потратил слишком много времени
думаю об этом, потому что я работал над новым интерфейсом
для Maxima (символьная математическая система), которая представляет собой программную
формы в математической нотации, а это значит, что я должен полностью
однозначный. Если вам интересно, вот демонстрационное видео.
Мой ответ:
спасибо за содержательные комментарии. Для меня это похоже, из Швейцарии,
где нас не учили явному правилу PEMDAS. Тем не менее, я бы все равно
утверждают, что миллионы людей были обучены PEMDAS. Я со всем согласен.
Особенно при работе с математическим программным обеспечением
потребность в точности и недвусмысленности становится все более важной.
Тема не только педагогическая, здесь можно допустить серьезные ошибки. Не только
делать неправильные вычисления, но также используя результаты, которые были
сказано неоднозначно.Было бы интересно узнать, сколько
студенты в Сингапуре или Швеции читают, скажем, 8/2 (2 + 2) как 16 и как
многие прочитали бы это как 1, особенно если разделение дано как обелус. я
предсказал бы, что большинство из них дадут ответ 1. Это потому, что я учу
часто студенты со всего мира (Этим летом также двое из Сингапура
кстати) и что вопросы о PEMDAS часто всплывают, если не
супер четкость при написании (даже при написании выражений типа 1 / x + 3 есть
все еще студенты, которые читают это как 1 / (x + 3) или сбиты с толку и спрашивают.я начал
чтобы даже прояснить такие вещи, написав (1 / x) + 3 или используя горизонтальный
обозначение дробей \ frac {1} {x}, чтобы убедиться, что оно правильно прочитано.
 

Добавлено 24 августа, 2020 :

Добрый день! Надеюсь, у тебя все хорошо.
Совсем недавно у нас с коллегами было несколько жарких споров.
о проблеме 8 ÷ 2 (2 + 2)
или тому подобное. Как и следовало ожидать, есть люди, которые отвечают
это как 16, а люди, которые отвечают на это как 1. Что касается нас, мы находимся в
неоднозначный стан.Люди, у которых был уникальный ответ на проблему, продолжают цитировать
PEMDAS, или BEDMAS, или PE (MD) (AS). И мы сказали им, что
условность не универсальна. Итак, мы попытались найти документы
чтобы поддержать нашу сторону. И вот мы нашли вашу статью. я прочел
все. Я поражен тем, что вы так терпеливо отвечаете
людям, которые отправляют вам электронные письма и дают им одинаковые ответы,
повторяя ваши объяснения снова и снова.

В любом случае, я только что написал вам по электронной почте, чтобы вы знали (на случай, если вы
еще не видел) Раздел 7.1.3, стр.23 ISO 80000-1: 2009 "
 
"Эти процедуры могут быть распространены на случаи, когда числитель или знаменатель, или оба они сами являются продуктами или частными. В таком сочетании солидус (/) не ставится. знаком умножения или деления на той же строке если не вставлены круглые скобки, чтобы избежать двусмысленности ".
Кроме того, пример 1, следующий за абзацем, должен разрешить все эти споры.
Спасибо, что нашли время прочитать мое письмо.
Мой ответ
Большое спасибо за отзыв.
Ссылка ISO очень ценна!
На это уже указывалось ранее, но, к сожалению,
эта часть ISO не является общедоступной. Нужно
купить стандартный.
 
У меня действительно был доступ к соответствующие страницы из Раздела 7.1.3 ISO 80000-1: 2009:

Добавлено 16 января 2021 г. : Электронная почта:

После бурного онлайн-обсуждения меня перенаправили на ваш сайт
http: // люди.math.harvard.edu/~knill/pedagogy/ambiguity/index.html.
Судя по тому, что я там читал, вы, кажется, вините путаницу в использованных
обозначение. Хотя я согласен с тем, что это может сбить с толку людей, я хотел бы
представить более универсальное объяснение этой проблемы, основанное на использовании
(или не используя) простые аксиомы алгебраических структур. Обозначения тогда
становится более или менее неуместным.

У данного уравнения 6: 2 (2 + 1) есть несколько ответов, которые
на первый взгляд кажется нелогичным. Но я думаю, что некоторые люди просто забывают
чтобы определиться с диапазоном определений, над которыми они работают.С простым
арифметика, умножение и деление одинаково упорядочены и имеют
просто для выполнения перед сложением / вычитанием, но после вычисления
скобки и показатели. Вот где появляются PEMDAS или другие мнемоники
из. Это работает слева направо и правильно само по себе
простой контекст. Калькуляторы последовательного ввода делают это таким образом.

Но если принять контекст формальных алгебраических структур, в
В этом случае поле рациональных чисел, применяются дополнительные аксиомы. Теперь,
«распределенность» и «ассоциативность сложения и умножения»
(среди прочего) определены как основные аксиомы в контексте этой области.При этом порядок операций теперь гораздо более ограничен теми, кто
характеристики.

В этом контексте уравнение 6: 2 (2 + 1) можно преобразовать следующим образом:

6: 2 (2 + 1) = 6: (2 + 1) 2 // Ассоциативное свойство умножения
(просто чтобы доказать свою точку зрения с помощью этой аксиомы поля): ab = ba; разделение
неассоциативен

= 6: (4 + 2) // Распределительное свойство: ab + ac = a (b + c)

= 6: 6 // Порядок операций (теперь тривиальный),
"скобки перед показателями перед умножением / делением перед
сложение / вычитание "

= 1


Соблюдая все аксиомы полевой структуры, разделение теперь должно быть
решается путем вычисления сначала делимого и стороны делителя отдельно, затем
частное.Любой другой способ нарушает одну из основных аксиом поля и
будет недействительным с точки зрения этого поля. Более сложные калькуляторы
запрограммированы уважать это.

Однако, если не ограничиваться контекстом этих аксиом, другие результаты
также может быть действительным (т.е. 6: 2 (2 + 1) = 9, с последовательным вычислением на
базовый арифметический уровень).

Так что, на мой взгляд, проблема не в нотации, а в предоставлении контекста.
и, таким образом, определяющие аксиомы. Знание используемых инструментов и их предположений
соответствующие способности / ограничения - как всегда - важны.Кстати, в Германии студенты изучают мнемонику «KLAPPS» (KLAmmer,
Potenz, Punkt, Strich) перед высшей алгеброй, которая, к счастью, игнорирует
любое предлагаемое предпочтение умножения / деления ("Punktrechnung"
= "точечные вычисления" на основе "точечных" символов, используемых в немецком языке
вычисление '.' = Unicode U + 22C5 для умножения и ':' для деления)
и сложение / вычитание ("Strichrechnung" = "вычисления строк", из
символы «линия / перечеркнутая линия» '+' и '-'). Это отсутствие последовательности
предложение, кажется, предотвращает путаницу в высших классах, по крайней мере,
немного.Большое спасибо за то, что уделили время, прочитав это. Я был бы признателен услышать
от вас, особенно если вы не согласны с моими выводами. Математика
конечно не моя сильная сторона, но я просто учусь этому снова
с домашним обучением моего сына (благодаря изоляции COVID19) и
до сих пор помню некоторые из моих прежних лекций по информатике и математике
давно.

С наилучшими пожеланиями из Баварии / Германии! Бернхард Новотны, M.Eng.
 
Мой ответ:
Это интересный ракурс. Мне особенно приятно слышать о
немецкое правило KLAPPS.Я ходил в школу в Швейцарии и еще не
видел это. Правила также должны соблюдаться со студентами, и KLAPPS - это
хорошее имя. Интересно, что он не решает двусмысленность PAMDAS.
Ваше предложение устранить двусмысленность вполне может сработать, но
проблема более социолингвистическая, чем математическая. Уже есть
разные стандарты вокруг. Я не знаю, как вводить новый
XKCD сказал это лучше всего.
 
22 февраля 2021 г .: Вик:
Вы писали, что PEMDAS и BODMAS неоднозначны. Я не согласен.У меня есть степень магистра математики и
преподаю математику 30 лет. Вы задали вопрос с помощью символа разделения /.
Мне кажется, что этот символ разделения сбивает с толку, и я никогда его не использую.
Мне интересно, что хотя все калькуляторы дали ответ 11 на ваш
начальная проблема вы, кажется, думаете, что они должны что - выдать сообщение об ошибке?
Ответ - 11. Здесь нет скобок, и я не понимаю, почему вы думаете
вы можете вставлять скобки волей-неволей.
Люди ошибаются, потому что они вычисляют по шаблонам, а не думают.
«Какие здесь правила».
Мой ответ:
Спасибо за ответ. Вы, конечно, можете не согласиться. Я дал много
причины на моей странице, почему есть двусмысленность. Многие студенты дают
ответ 2, а не 11 в исходном примере. Если вы прочитаете мой текст, вы увидите
эта двусмысленность не означает наличие сообщения об ошибке. В большинстве случаев,
несогласие происходит из-за отсутствия стандарта. Профессиональные ссылки
приведенные на моей странице подтверждают, что нет стандарта. Ставить скобки нельзя
помещенные «волей-неволей», они размещены, чтобы прояснить ситуацию и устранить двусмысленность.Во время обучения почему бы вам не провести эксперимент и не спросить студентов, что они
подумайте, когда им нужно вычислить 2x / 3y -1 для x = 9 и y = 2. Большинство людей дает
ответ 2, поскольку они заключают в скобки 3y как неразделимые (довольно много учителей подтвердили
который). Это также стандарт, которому следуют многие книги (особенно если они придерживаются
к более ранним профессиональным стандартам, таким как AMS, или правилам, таким как PEMDAS, которые
часто учили и ставили умножение перед делением. Коджори, сегодня по-прежнему самый
Авторский деятель по математике примечания написал еще в 1928 г.
"Если арифметический или алгебраический член содержит / и х, в настоящее время
нет соглашения о том, какой знак должен использоваться первым."Это актуально и сегодня
Оливер
 
и продолжение Вика
PEMDAS не ставит умножение перед делением. Они идут по порядку
слева направо. Просто потому, что ученики соединили 3й как неразлучные
не означает, что ученики правы. Они делают предположение. .
Какие книги нравятся студентам? Я никогда не видел ни одной из этих книг.
/ означает деление. Другого толкования нет. Так ты говоришь
что PEMDAS не является общепринятым правилом? Так выражение вроде
2/3 + 5/8 x 7 может иметь много разных ответов в зависимости от того, где вы положили
скобки?
 
Мой ответ:
Вторую часть можно было бы прочитать как 5/56 и получить 127/168, в то время как компьютеры
прочтите это как 2/3 + (5/8) 7 = 121/24 (на самом деле не все компьютеры, но
большинство поколений калькуляторов давали другие результаты). 2.Ты
скажем, вы делите торт на 2 человек, имея в виду торт / (2 человека), а не
(торт / 2) чел. Часто, особенно для установленных формул,
мы естественно берем их вместе. Популярный пример - 1 / 2pi, потому что
2pi часто сам по себе является стандартным устройством. Большинство людей читают это как 1 / (2pi)
но компьютер читает это как пи / 2. Бывший 1 / (2pi) соглашается с написанием
руководящие стандарты, выдвинутые AMS или APS.
 
Снова Вик:
Если 2x / 3y означает 2x / (3y), как нам написать выражение 2 раза x
затем разделить на 3, а затем умножить на y?
 
Мой ответ:
Пример 2x / 3y - типичный пример, который большинство людей читают как (2x) / (3y).Компьютер читает это как 2xy / 3. Можно написать (2xy) / 3 или (2/3) xy или просто 2xy / 3
если бы кто-то хотел, чтобы это было написано так, как это читает компьютер.
Это не двусмысленно и в целом, и никто не может его неправильно истолковать.
 
23 февраля 2021 г., Джеймс:
Мне очень нравится ваша статья (я бы назвал ее так). Это
привлекла мое внимание, потому что я начала видеть работы студентов
которые вставляют в свои калькуляторы целые выражения, отражающие
несоответствия, обсуждаемые в вашей статье.Одна вещь, которую я отмечу, касается примера 18/3 * 2, чтобы
в котором вы утверждаете, что большинство людей придут к решению 3,
а компьютеры - 12. Сразу после того, как увидели выражение,
Я рассчитал, что решением будет 12. Я считаю, что прочитал это.
как восемнадцать третей умножить на два. Я списываю это на то, что часто работаю
с четвертями, третями и половинами вне работы и, когда я вижу
"/" Я сразу думаю о дроби, а не об операции. Более
доказательства в поддержку вашего аргумента о двусмысленности в математике и
необходимость ясности в выражениях.Еще раз спасибо за приятное чтение. я в предвкушении
исследуя остальную часть вашего сайта.
 
Мой ответ:
Спасибо за ответ. Да, это очень интересная история. Это
кажется, также во многом зависит от того, как это писать. Часто в
научная литература (а также некоторые руководства от AMS или APS),
указано, что такие вещи, как a / bc, следует читать как a / (bc)
а не (a / b) * c. Причина также в том, что часто товары читают
люди должны принадлежать друг другу. Если взять 18 / 2π, то большинство людей
прочтите это как 9 / π, а не как 9π (как это делает компьютер).Только можно
догадываюсь, но я подозреваю, что это было причиной создания PEMDAS (наиболее
часто используемое сокращение), а не PEDMAS или PE (MD) AS (которые должны быть
со сноской, что MD эквивалентны и читаются слева).
Последнему вряд ли можно научить. PEMDAS уже трудно продать
(многие студенты Гарварда неправильно понимают базовые PEMDAS и читают +
перед *. В педагогических вопросах всегда переоценивают то, что
сложности, которым люди могут научиться. Устанавливаем новый стандарт, такой как PE (MD) AS
(они появляются уже в школах) также имеет проблему, которая
многие тексты будут прочитаны неправильно.Если выражение принадлежит друг другу
как RT в термодинамике или mc  2  в физике. Вернемся к вашему примеру:
18/3 * 2, да, думаю, многие прочитали бы это как 12, но если вы напишете
18 / ab с a = 3, b = 2, большинство прочитало бы это как 3. Забавно, как по-человечески
Здесь сочетаются психология, лингвистика и история, а также математика.
Это делает тему такой интересной.
 

28 февраля 2021 г. : хороший новый пример:
Я учитель математики в небольшой католической школе.Недавно я столкнулся с прикрепленной проблемой (# 12)
в моей программе спирального обзора в шестом классе. Это похоже на вопрос, который вы задаете, но в нем используется знак деления
вместо  "/". Я разместил его в группе учителей математики, в которую я участвую, для средней и старшей школы по математике и СВЯТОЙ КОРОВЕ!
Это привело к дебатам / спорам. Есть те, кто непреклонен, что ответ - 6, те, кто непреклонен.
что ответ 24, и те, кто непреклонен, что ответ может быть любым.
Конечно, каждая группа думает, что они правы!
Ваша статья была размещена в ветке несколько раз.Я прочитал подавляющее большинство из них. Мне любопытно, если
вы видите разницу в неоднозначности прилагаемой проблемы с разницей в использовании символов или
если это останется такой же загадкой, как и проблема, которой вы поделились.
Большое вам спасибо за ваше время!
 

Мой ответ:

Спасибо. Это очень ценно, потому что это подтверждает, что
единственный способ избежать таких обсуждений - это быть предельно ясным и добавить
кронштейны. Добавил в коллекцию.Хорошо то, что все
в вашей группе учителей правы. Есть веские аргументы в пользу группы 2c
вместе, потому что это часто используется в литературе, там
веские аргументы в пользу разделения в первую очередь, потому что это то, что
делать. Кроме того, есть веские аргументы в пользу того, что оба правы. Но кто когда-нибудь выиграет
аргумент все равно столкнется с дилеммой оценить это ...
буря расстроенных учеников, родителей и других учителей. Я просто буду
охватываю алгебру в моем учебном курсе по математике и могу упомянуть об этом
 

4 марта 2021 года : обсуждения в социальных сетях, похоже, все еще продолжаются: Я получил следующее электронное письмо:
Мне было интересно узнать об этом уравнении.2. Если это так, ПЕМДАС ГОВОРИТ «Экспоненты» - 2-й. Так что мой
вопрос действительно в том, что экспоненты также являются формой умножения, так почему это
теперь M такое же, как D в уравнении, где теперь он заменен на PE (M или D
слева направо) (A или S слева направо)? В мои математические годы умножение
в-третьих, потому что экспонента и умножение одинаковы. Также когда мы
упрощать 6/9 упрощается как 2 (3) / 3 (3), мы не можем упростить это как ответ
6, когда должно быть 2/3. Вот новинка в социальных сетях: 24/4 (8/4)
и мой ответ = 3.Некоторые говорят, что это 12. После того, что я объяснил
над. Какой твой ответ? Заранее спасибо.
 
Мой ответ:
Корпус 6/2 (1 + 2) теперь классический. Большинство людей получают 1. Это не потому, что
это целое число, но потому что 2 (1 + 2) рассматривается как единица. Наиболее
компьютеры получают 9, потому что (M и D) оцениваются на одном уровне.

Новых 24/4 (8/4) не видел. У вас есть 3, что получает большинство людей.
Миллионы из них, потому что это закреплено в правилах PEMDAS (M перед D)
и потому, что это часто пишется в книгах как таковых.Большинство компьютеров получают 12.

Здесь нет правильного или неправильного ответа. Вопрос только что поставлен
неоднозначно. Это известно уже 100 лет. это
Интересно, что соцсети до сих пор об этом гудят. Но это делает это
интересно и осведомленно. Я написал краткое резюме с источниками в
этот документ
для курса, который я преподаю прямо сейчас
 
Очевидно, что на гораздо более базовом уровне все неясно. Следующее электронное письмо иллюстрирует замешательство, которое может возникнуть даже у звездных студентов. Кажется, сложно даже достать через фундаментальные свойства PEMDAS, такие как возведение в степень перед умножением и разделение и скобка, перед которыми не оспариваются.2 = 1, то это однозначно, поскольку возведение в степень предшествует другим операциям. Если написать 9/3 (3), то это неоднозначно, потому что его можно прочитать как (9/3) (3) = 3 * 3 = 9 или 9 / (3 (3)) = 9/9 = 1. Случай 6/1 (1 + 2) - неоднозначный случай, потому что его можно читать как (6/1) (1 + 2) = 18 или 6 / (1 (1 + 2)) = 2. Это та ситуация, о которой мы говорили. Нет, 6 / (1 + 2) не является неоднозначным. Ясно, что 6/3 = 2. Нет обсуждения об этом случае, потому что скобки сделаны раньше.

13 марта 2021 г .:
По профессии я технический писатель, поэтому лаконичное, точное общение - это то,
моей страсти.Я также изучал математику в бакалавриате, так что это
обсуждение было прямо моим союзником. Спасибо за поддержку этой страницы. Это было приятное путешествие.

Как упомянул один из рассылающих по электронной почте, где это возможно, я предпочитаю наборные движки, например Латекс.
Почему бы никому не написать $ \ frac {2x} {3y} - 1 $ (или что-то подобное), если бы они
наличие выбора утомительно для меня, но это часть философии моей профессии.
Я беру время и когнитивные усилия, чтобы четко объяснить что-то, чтобы свести к минимуму
усилия, которые читатели должны приложить, чтобы понять это.В отдельном электронном письме упоминалось использование знака деления или обелуса: (LaTeX: $ \ div $),
что заставило меня понять, что я не оцениваю и / так же. Назначаю разные психические
вероятности к разным интерпретациям. Если бы мне вручили документ,
содержит ab (c), я бы сначала проклял автора и начал поиск контекстных подсказок. Если я
не нашли никаких подсказок, я бы отметил, что они нашли время, чтобы использовать персонажа вне основного
Набор ASCII. Это говорит мне, что они, возможно, а может быть, намеренно избегали
пресловутый, очевидно неоднозначный слэш.Поэтому я думаю: «Что бы наиболее распространенное,
Самая ленивая интерпретация этого уравнения была бы, если бы автор использовал косую черту? "
Вероятно, это то толкование, которого они пытались избежать.

Иногда мне доступен один кусочек контекста, когда я могу решить, сколько усилий они приложили
Чтобы избежать косой черты, они используют то устройство, которое они используют. Ввод текста в macOS
тривиально (Alt + /), но для набора текста на iPhone требуется либо копирование и вставка, либо
переход на отдельную клавиатуру стороннего производителя.

Так что это мой вклад! Это больше ориентированный на поведение подход: количество усилий, которые он
потребовалось, чтобы автор напечатал что-то, что может предложить их предполагаемое значение.С уважением,
Ник
 
Мой ответ:
Привет, Ник,
приятные моменты. Для меня также это то, что когда я пишу латексные формулы
в тексте и не отображается, я склонен использовать a / bc вместо \ frac {a} {bc}
потому что он лучше вписывается в страницу. Поскольку я знаю об этих войнах PEMDAS, я
Я больше не сомневаюсь и пожертвую красотой, если она может увеличить ясность. Этот
для меня как учителя особенно важно на экзаменах.

Повсюду используются знак деления, обелус и косая черта. Я также узнал,
особенно от учителей, что для них обелус используется иначе и
что это более сильное подразделение.Я сам стараюсь избегать обелуса.
Я знаю только, что видел это в начальной школе и не прикасался к нему
поскольку.

Да, ясность требует некоторых усилий, иногда также требуется жертва некоторых
элегантность. Но мы знаем это и по языку. Есть много выражений
которые становятся понятными только в контексте. Ответ однозначно - избыточность.
Даже язык нашего генома использует избыточность, чтобы избежать
ошибки связи.

Оливер
 

строк, целых чисел и чисел с плавающей запятой

В этом руководстве мы собираемся начать работать с основными типами Python: строки (для текста) и целые числа и числа с плавающей запятой (для числовых значений).

Обратите внимание, что в этом руководстве вы работаете в REPL (IDLE). Вы можете найти дополнительную информацию о REPL и о том, как запустить Python в вашем cmd или терминале в руководстве по установке Python.

Привет, мир!

По традиции мы начнем с печати «Hello, World!» к консоль. В Python функция для достижения этого метко названа print () . Введите следующее рядом с >>> :

REPL просто напечатает текст прямо у вас.

Теперь напечатайте свое имя и немного поэкспериментируйте!

Несколько аргументов

Еще кое-что интересное в функции print () заключается в том, что вы можете передать ее несколько аргументов для печати:

  >>> print («Привет», «До свидания»)
  

Фактически, вы можете передать в печать столько информации, сколько захотите:

  >>> print («один», «два», «три», «четыре», «пять», «шесть», «семь», «восемь», «девять»)
  

Конечно, вы также можете получить такой же результат с помощью этого:

  >>> print («один, два, три, четыре, пять, шесть, семь, восемь, девять»)
  

Вам решать, что подходит и когда.

Математика

Любимое времяпрепровождение.

Простая арифметика

Python может выполнять простую арифметику. Для начала попробуем сложить:

Теперь вы должны увидеть результат этого вычисления в REPL.

Вычитание, умножение и деление работают одинаково.

  >>> 6 - 2
>>> 8 * 4
>>> 9/3
  

А теперь попробуйте еще несколько, чтобы увидеть, какие результаты вы получите. Попробуйте свои силы во всех основные математические операторы: + , - , * (умножение), / (деление), ** (экспоненты) и % (модуль).

Объединение операций

Возможность выполнять только одну операцию за раз довольно ограничивает, поэтому Python позволяет нам комбинировать математические операции. Попробуйте это:

А теперь попробуйте еще несколько. Вы можете комбинировать столько операций, сколько захотите.

Приоритет оператора

Заметили ли вы неожиданные результаты, когда начали комбинировать операции? Если вы этого не сделали, попробуйте это:

Python следует традиционным математическим правилам приоритета, которые гласят: это умножение и деление выполняются до сложения и вычитания.(Ты может вспомнить BODMAS .) Это означает, что в В нашем примере выше сначала умножаются 2 и 4, а затем результат вычтено из 10.

Мы можем изменить порядок операций, используя круглые скобки. Что-нибудь внутри круглые скобки выполняются первыми.

А теперь попробуйте вот так:

У вас должен быть другой ответ.

Из-за правил приоритета сложные операции, такие как наш первый пример, могут будет довольно сложно читать. Если вы обнаружите, что пишете более сложный выражений, нет ничего плохого в добавлении круглых скобок для ясности.

десятичных знаков

Одна из вещей, которая сбивает с толку новичков в программировании в целом, — это концепция чисел с плавающей запятой . В основном числа с десятичными точки имеют тенденцию вести себя немного странно, когда вы выполняете математические операции на них. Причины этого сложны и коренятся в природе. вычислений, поэтому пока давайте просто понимаем, что с десятичными числами происходят странные вещи.

Чтобы увидеть пример этого, попробуйте разделить 10 на 3 :

Ответ должен длиться вечно, но это не так.А теперь попробуйте что-нибудь немного более чувствительный к точности:

  >>> 1.000000000000001 * 8
  

Наверное, не то, что вы ожидали, верно? А пока тебе просто нужно принять это в качестве ограничения, и позже вы узнаете, как другие программисты работают с Это.

Заключение

Теперь давайте объединим то, что мы узнали сегодня. Мы можем сказать print () для печати сразу несколько вещей, разделенных запятой:

  >>> print ('Результат 2 + 2 равен', 2 + 2)
  

Сохраните свою работу

В этом уроке вы кодировали REPL (IDLE), но много раз вы хотите чтобы вместо этого сохранить ваш код.В таких случаях вы можете сохранить свой код в файл с помощью текстового редактора. Мы даем некоторую информацию о текстовых редакторах в нашем Руководстве по началу работы.

Откройте текстовый редактор и напишите код из первого упражнения:

Сохраните файл как ex1.py . Вы можете называть файлы как хотите, но они должны заканчиваться на .py , чтобы питон мог их легко прочитать. Читая ваш файл в Python, вы снова будете использовать оболочку cmd или terminal . Вы можете прочитать свой файл с помощью следующей команды (введите без знака $):

Если перед кодом указано >>> , значит, вы все еще находитесь в REPL (IDLE) и вам нужно выйти. это с:

Тогда вы сможете загрузить файл.

На этом мы завершаем сегодняшнее руководство. В следующем уроке мы узнаем, как объединить результаты нескольких отдельных выражений с использованием переменных, получить ввод от пользователя и принимать решения на основе этой информации.

Дополнительная литература

В Руководстве разработчика Google есть очень хорошая вводная статья.

Вы также можете найти ресурсы для начинающих на веб-сайте Python. и обратитесь к документации Python, где объясняются основы языка.

Вернуться к обучающим материалам главная страница кодовой панели

Разъяснение правила PEMDAS! (Примеры включены) — Mashup Math

P: Круглые скобки

E: Экспоненты

M: Умножение

D: Деление

A: Сложение

S = Вычитание

Операции, указанные в правиле PEMDAS, выполняются слева направо.

Кроме того, правило PEMDAS для вызова математического порядка операций имеет несколько важных подправил, которые также необходимо соблюдать, если вы хотите правильно использовать PEMDAS (и получать правильные ответы на математические задачи).Эти важные подправила относятся к отношениям между умножением / делением и сложением / вычитанием.

Эти важные подправила правила PEMDAS подробно объясняются в следующем разделе:

Правило PEMDAS: ключевые моменты

Правило PEMDAS существует уже несколько десятилетий как инструмент, помогающий учащимся запомнить математический порядок операций. . Многие предпочитают просто запоминать мнемоническое PEMDAS (произносится PEM-DAHS), в то время как другие предпочитают запоминать фразу Пожалуйста, извините мою дорогую тетю Салли.

Однако вы решили помнить, что правило PEMDAS не так важно, как запоминание ранее упомянутых подправил? Почему так важны подправила правила PEMDAS? Потому что вспомогательные правила часто определяют разницу между получением правильного или неправильного ответа на математическую задачу.

Правило PEMDAS может быть несовершенным, но если вы помните вспомогательные правила, оно может быть полезным инструментом, который поможет вам правильно применять математический порядок операций и получать правильные ответы как на простые, так и на сложные математические задачи при условии, что вы знаете важных подправила.

Важные подправила к правилу PEMDAS:

1.) P: Выполняйте операции внутри скобок или групп, прежде чем делать что-либо еще (если нет групп или скобок, вы можете пропустить этот шаг) .

2.) E: Затем, после выполнения операций внутри скобок и группировок (если они есть), примените любые экспоненты (если нет показателей, вы можете пропустить этот шаг).

3.) M / D: Затем, после скобок, групп и экспонент, выполните умножение / деление слева направо в зависимости от того, какая операция будет первой).

★ Тот факт, что M стоит перед D в правиле PEMDAS, не означает, что вы всегда будете выполнять умножение перед делением.

4.) A / S: Наконец, после умножения и / или деления выполните сложение / вычитание слева направо в зависимости от того, какая операция будет первой).

★ Тот факт, что A стоит перед S в правиле PEMDAS, не означает, что вы всегда будете выполнять сложение перед вычитанием

= Чрезвычайно важно

Почему учителям математики пора отбросить BODMAS

Что означает BODMAS?

Акроним BODMAS означает скобки, порядки, деление, умножение, сложение, вычитание.

Иногда его называют BIDMAS (с «индексами» вместо «заказов») или правилом PEMDAS в Америке (с «круглыми скобками» и «экспонентами»).

Правило BODMAS

Это математическое правило диктует правильный порядок операций, которым нужно следовать, когда вы заполняете вопрос с математическим числовым предложением с различными операциями.

Первый шаг — сделать что-нибудь в скобках, затем заказать следующие (например, извлекать квадратный корень или индексы). Деление и умножение находятся на одном уровне, что означает, что им дается равный приоритет, и они должны выполняться слева направо, а не все деление, а затем все умножение.Точно так же сложение и вычитание находятся на одном уровне и должны выполняться слева направо.


Я начал свою педагогическую карьеру в средней школе. Молодой, нетренированный и еще не лысеющий, я оказался на самом крутом этапе обучения в моей жизни.

Еженедельные встречи с моим руководителем отдела были жизненно важны для обсуждения педагогики, и я строго придерживался его инструкций: «Никогда не сокращайте совокупную частоту», «Мы всегда подбрасываем монеты и получаем решку, мы никогда не подбрасываем монеты и не получаем орла», и что очень важно. , «Мы никогда, никогда не используем БОДМЫ».

Отказаться от использования BODMAS оказалось труднее, чем вы думаете. Приехали студенты, хорошо разбирающиеся в его применении.

Нам пришлось и научить этому. Нам приходилось убеждать комнаты, заполненные подростками, в том, что они должны изменить основные принципы своей арифметической системы убеждений. Это было сложно, потому что подростки ненавидят перемены и ненавидят прозелитизм взрослых.

Так зачем нам вообще беспокоиться? Что убедило весь отдел в том, что нужно приложить столько усилий для решения такого, казалось бы, тривиального вопроса?

BODMAS ошибается.Это то что.

Неправильный ответ

Буквы обозначают скобки, порядок (что означает степень), деление, умножение, сложение, вычитание. Таким образом, предполагается, что в этой последовательности происходит упрощение любого данного математического выражения.

Например, чтобы оценить 3 + (3 + 3) 3 ÷ 3 — 3 x 3 , мы действуем в указанном выше порядке:

Это был бы действительно полезный алгоритм, если бы он работал в любой ситуации, но рассмотрим гораздо более простое выражение 1 — 2 + 4 .Он не содержит скобок, степеней, деления или умножения, поэтому мы будем следовать BODMAS и выполнять сложение с последующим вычитанием:

Это ошибочно. Правильное значение — 3. BODMAS нас подвел. Позор БОДМАМ!

Математические задачи

У нас не может быть волшебной мнемоники, которая не работает все время; Предположим, он решил не работать в важный момент. Представьте, что вы пытаетесь объяснить своему ученику, что причина, по которой он потерял оценку на экзамене, заключалась в том, что то, что вы сказали ему, всегда работает, на самом деле не сработало во всех случаях, и, фактически, один из таких случаев произошел тот документ GCSE.

Это не новая проблема. Я не первый, кто об этом пишет. Даже Википедия решает эту проблему и предлагает несколько альтернатив. Студенты любят Википедию! Так почему же BODMAS все еще актуален?

В Хайгейте вокруг него было такое клеймо, что некоторые партии высмеивали меня более десяти лет после того, как мой коллега пережил обмен в классе, который проходил примерно так:

Учитель: Как нам упростить это выражение?
Студент: БОДМАС, сэр.
Учитель: Мы здесь не используем БОДМЫ.
Студент: Но вот чему мистер Элтон научил нас в прошлом году, сэр.

После этого мне несправедливо присвоили прозвище «БОДМАС», которое преследовало меня повсюду. У меня не было защиты; Заявление подал платный студент, так что оно должно быть правдой. По крайней мере, один человек (он знает, кто он) все еще называет меня БОДМАСом чаще, чем он использует мое настоящее имя.

Несмотря на то, что я абсолютно не виновен в том, что запятнал умы невинных студентов, я чувствую себя обязанным исправить положение, поэтому я использую эту платформу именно так.Считайте это общественными работами.

Правильный ответ

Нет смысла бить BODMAS, не предлагая альтернативы. Проиллюстрированная выше ошибка вызвана тем фактом, что сложение и вычитание не обязательно должны происходить в таком порядке. Если у нас есть строка этих двух операций, она называется суммой, и мы должны работать слева направо:

Точно так же деление не более важно, чем умножение. Если у нас есть строка из этих двух операций, она называется продуктом, и мы снова будем работать слева направо:

Теперь у нас такой порядок: скобки, порядок, продукты, суммы.

Это дает нам BOPS, который на целый слог короче, чем BODMAS, и имеет значительное преимущество в том, что он надежен.

Я уверен, что если бы кто-то предложил BOPS до BODMAS, то последний был бы предан безвестности. Даже сейчас еще не поздно избавиться от двусложных арифметических сокращений.

Я призываю своих коллег по всему миру запретить BIDMAS и очистить PEMDAS. Не оставляйте от них никаких следов. Позвольте BOPS нанести победный удар молодым математикам во всем мире.


Оуэн Элтон — учитель математики, автор / исполнитель глупых песен и автор математических минут. Вы можете следить за ним в Твиттере по адресу @owenelton.

Как решить математическую задачу с помощью PEMDAS

Обновлено 15 декабря 2020 г.

Крис Дезил

Взгляните на следующее равенство:

x = 7 + 2 × (11-5) ÷ 3

Решите для x , выполняя математические операции в порядке слева направо, и вы получите 18, что является неправильным ответом.Чтобы получить правильный ответ — 11, вы должны соблюдать правильный порядок действий. Если вы не можете вспомнить правильный порядок, PEMDAS может вам помочь. Это аббревиатура, обозначающая круглые скобки, экспоненты, умножение, деление, сложение, вычитание.

В общем, PEMDAS не так уж сложно запомнить, но если вы не можете это сделать, вам может помочь пара крылатых фраз. Один из них — «Прошу прощения, моя дорогая тетя Салли». Первая буква в каждом слове этой фразы является одной из букв в PEMDAS.Если вы предпочитаете называть круглые скобки скобками, вспомните аббревиатуру BEDMAS и крылатую фразу «Большие слоны уничтожают мышей и улиток». В этой фразе D и M меняются местами, но это нормально. Когда вы доходите до умножения и деления, вы обычно делаете то, что идет первым в выражении.

Некоторые люди, которым сложно запомнить PEMDAS, ищут порядок операций с помощью поиска PADMAS math. Это не поможет. Он игнорирует E для экспонент, а экспоненты — важная операция, которую необходимо выполнить, прежде чем вы перейдете к любой из других арифметических операций.

Как применять порядок операций

Всякий раз, когда вам нужно выполнить длинную цепочку операций, правила математики ясны. Вы всегда начинаете с выполнения операций в круглых скобках (скобках), а затем решаете экспоненты, которые представляют собой числа в форме x a . Следующие две операции — это умножение и деление. Если деление идет первым в выражении, вы делаете это в первую очередь. Точно так же, если сначала идет умножение, сделайте это в первую очередь.То же верно и для двух последних операций — сложения и вычитания. Выполняйте вычитание перед сложением, если они идут первыми в выражении, и наоборот.

Пример расчета

Еще раз взгляните на выражение в начале этой статьи. Применяя PEMDAS, вы решаете это следующим образом:

    11-5 = 6

    Таким образом, выражение теперь становится

    x = 7 + 2 × 6 ÷ 3

    Сначала идет умножение, поэтому начнем с этого.Теперь выражение выглядит следующим образом:

    x = 7 + 12 ÷ 3

    Теперь деление должно получиться:

    x = 7 + 4

    Необходимо выполнить только одно сложение, которое дает окончательный ответ:

    x = 11

    Иногда вы видите несколько скобок или скобок. Правило состоит в том, чтобы упростить все внутри скобок, начиная с внутренних, прежде чем переходить к остальным арифметическим операциям. Не забывайте следовать PEMDAS или BEDMAS даже при работе с числами в скобках.2 × 4

    • E — Решите все экспоненты:

    4 + 4 × 4

    • M, D — Произведите умножение и деление:

    4 + 16

    • A, S — Выполните сложение и вычитание:
    .

admin

Добавить комментарий

Ваш адрес email не будет опубликован. Обязательные поля помечены *